Explanation

 

11/20/24

 

Recognized today as one of the world’s leading authorities on the Sphinx, archaeologist, Mark Lehner, has conducted field research on the pyramids at Giza during most of the 37 years since his first visit to Egypt.

 

A. NO CHANGE
B. Sphinx; archaeologist, Mark Lehner
C. Sphinx, archaeologist Mark Lehner
D. Sphinx archaeologist Mark Lehner,

 

Although the question appears to be testing the punctuation after the word Sphinx and around Mark Lehner’s name, in reality, the various combinations of commas around Mark Lehner indicate that this question is really testing names with commas — in fact, it is unnecessary to consider the punctuation after Sphinx at all.

When commas are tested with names, there are really only two options: two commas (non-essential) or no commas (essential). The other options are just distractors. When the name is crossed out, the sentence does not make sense: Recognized today as one of the world’s leading authorities on the Sphinx, archaeologist…has conducted field research on the pyramids at Giza during most of the 37 years since his first visit to Egypt. The name is clearly essential to the meaning of the sentence, so no commas are required. That makes the answer (C).

 


 

11/19/24

 

Coffee rust has plagued farmers for more than a century. When a tree gets infected by it, its leaves produce a brown, thin powder when scratched, pretty much like iron rust. The disease, caused by the fungus Hemileia vastatrix, also de-colors the bush’s leaves from a bright green to a brownish yellow. In the end, the tree loses all its leaves, as well as its ability to produce beans.
Coffee plants flourish in soil that is low in acid but high in nitrogen. In the late 19th century, Sri Lanka, the Philippines, and other countries in Southeast Asia were the world’s major exporters of coffee, but in a matter of decades, their coffee industries were nearly destroyed.

 

Which of the following provides the best transition to the information that follows?

 

A. NO CHANGE
B. If allowed to spread, the disease can have dramatic consequences.
C. Temperatures in countries outside the tropics are too variable for coffee trees to thrive.
D. Coffee rust typically infects plants grown at elevations below 5,000 feet.

 

Although the question asks you to identify the best “transition,” in reality you need to focus on the information that comes after because it must logically follow from the underlined statement. What is the focus of that information? That the coffee industries were nearly destroyed in several countries — since the passage is about coffee rust, it can be assumed that coffee rust was the culprit. (B) sets up that information most effectively: the statement that “the disease (i.e., coffee rust) can have dramatic consequences” is directly consistent with the idea of an entire industry being destroyed. (D) mentions coffee rust but does not focus on its destructive abilities. (A) and (C) are simply off-topic.

 


 

11/18/24

 

Native to South America, the cashew plant was brought by the Portuguese to India around 1560 and had spread east into Asia and south into Africa during the seventeenth century.

 

A. NO CHANGE
B. has spread
C. spread
D. will spread

 

(A) is incorrect because it contains the past perfect: had + verb. When a sentence refers to two finished actions, the past perfect can be used for the action that came first. In this case, however, the underlined verb refers to the action that came second. Action 1: the cashew plant was brought to India around 1560. Action 2: it spread into Asia and Africa during the seventeenth century.

(B) is incorrect because it contains the present perfect (has + verb). This tense is used only to refer to actions that are continuing into the present, but the dates 1560 and seventeenth century indicate that both actions in the sentence occurred in the past.

(D) is incorrect because will + verb is used to refer to future actions, and again, both actions are in the past.

(C) is correct because spread is in the simple past tense and is parallel to the other simple past tense verb in the sentence, was.

 


 

11/17/24

 

According to marine biologist Gil Rosenthal, distance and motion often makes it difficult for certain predators to perceive fine details on the bodies of their prey.

 

A. NO CHANGE
B. often make
C. has often made
D. often making

 

The sentence contains a compound subject (distance AND motion), which is plural, so a plural verb is required. (A) and (C) are incorrect because makes and has are singular — remember that plural verbs do NOT end in -s. (D) is also incorrect because the gerund making creates a fragment. That leaves (B), which correctly supplies the plural verb make. Note that the tense switch in (C) is a distraction — the question is testing subject-verb agreement only.

 


 

11/16/24

 

The architect Renzo Piano earned a reputation as an innovator for his design of Paris’s Pompidou Center. The Center’s exterior consisted of brightly colored tubes and marked a radical break with tradition.

 

What is the best way to join the sentence at the underlined portion?

 

A. Center, whose exterior consisted of brightly colored tubes that marked
B. Center, so its exterior consisting of brightly colored tubes marking
C. Center, its exterior consisted of brightly colored tubes that would mark
D. Center; and whose exterior consisted of brightly colored tubes that marked

 

Although each answer choice contains a fair amount of information, in reality, each of the incorrect answers can be eliminated on straightforward grammatical grounds.

(B) is incorrect because the gerund marking turns the second clause into a fragment. A clause begun by a conjunction (so) must contain a conjugated main verb rather than an -ING word.

(C) is incorrect because it contains a comma splice — two complete sentences separated by only a comma (tip-off: comma + its).

(D) can be eliminated easily because a semicolon should not be placed before the coordinating conjunction and. Any answer that contains this construction is incorrect.

(A) is correct because it appropriately uses the pronoun whose to refer back to the Pompidou Center (note that whose, unlike who, can refer to both people and things) and contains a main verb (consisted).

 


 

11/15/24

 

In a recent Job Outlook survey, employers rated the “ability to verbally communicate with persons inside and outside the organization” as the most important quality in perspective workers.

 

A. NO CHANGE
B. for perspective
C. in prospective
D. to prospective

 

Based on the context, the underlined section is referring to potential, i.e., prospective, workers. Perspective means “point of view,” and does not make sense in context. It also cannot be used as an adjective, as is the case here. That eliminates (A) and (B). (D) is incorrect as well because employers are looking for a quality that prospective workers possess — not a quality to them, but a quality in them. (C) is thus correct.

 


 

11/14/24

 

Reading fiction allows people to understand other people’s actions by entering into characters’ minds and seeing situations from their interior points of view. In fact, recent studies suggest that people, who read novels regularly, tend to have higher levels of empathy.

 

A. NO CHANGE
B. people who read novels regularly,
C. people, who read novels regularly
D. people who read novels regularly

 

The original version is both grammatically acceptable and incorrect. The problem is that the commas surrounding the clause who read novels regularly indicates that the clause is non-essential: that is, it can be removed from the sentence without altering the passage’s meaning. When the clause is removed from the sentence, however, we get this: In fact, recent studies suggest that people…tend to have higher levels of empathy. The sentence no longer specifies which people tend to have higher levels of empathy — and the sentence is not talking about people in general but rather one specific group of people (those who read novels regularly). The clause is therefore essential to the meaning of the sentence, so no commas should be used. That makes the answer (D). Note that in a question testing non-essential clauses, there are really only two options: two commas or no commas. It is never correct to place a comma only at the beginning or end of a non-essential clause, so (B) and (C) can be eliminated.

 


 

11/13/24

 

Dinoflagellates emit blue light when disturbed, which is why they can be seen sparkling over wave crests, around boats or when a hand or paddle runs through them. These tiny creatures are the most common source of bioluminescence at the ocean’s surface. So-called bioluminescent bays such as in Puerto Rico and Jamaica are among the best-known places to witness the glow. However, the ephemeral phenomenon can be found throughout the ocean where there are dense gatherings of dinoflagellates. Sometimes dinoflagellates’ population increases rapidly, causing blooms, which by day are coloured a less attractive red-brown, sometimes known as red tides. And some, but not all, of these red tides are poisonous.

 

The passage implies that a rapid increase in dinoflagellates

 

A. can be harmful to other creatures.
B. improves dinoflagellates’ ability to produce light.
C. occurs primarily in warm waters.
D. is associated with the emission of blue light.

 

Although the passage contains a fair amount of information, in reality you only need the last two sentences to answer the question. But because this is an inference question, the answer is not stated word-for-word. The phrase Sometimes dinoflagellates’ population increases rapidly indicates that the necessary information will follow. The passage then goes on to state that the rapid increase in dinoflagellates’ population causes red tides, and that some, but not all, of these red tides are poisonous — by definition, something poisonous “can be harmful to other creatures.” That makes the answer (A).

Be careful with (B): the passage states only that [dinoflagellates] are the most common source of bioluminescence. It does not at all indicate that a rise in their population is in any way connected to their light-production ability. In (C), don’t get thrown off by the reference to Puerto Rico and Jamaica. It is reasonable to assume that these islands are surrounded by warm waters, but this discussion is located in the wrong part of the passage and is completely off-topic. (D) is off-topic as well.

 


 

11/12/24

 

Ammonia is a waste product that can be toxic to animals, however, plants, including phytoplankton, prize ammonia as the most energy-efficient way to build new cells.

 

A. NO CHANGE
B. animals however
C. animals however,
D. animals, but

 

When however is used to begin a clause, as is the case here, it must follow a semicolon. There is no option here that includes that construction, making (A), (B), and (C) incorrect. (D) solves the problem by using but, which can acceptably follow a comma.

 


 

11/11/24

 

Hanging low on the horizon, ancient Polynesian mariners were helped by bright stars to navigate between the many islands of the Pacific Ocean.

 

A. NO CHANGE
B. Bright stars, which acted as compasses that helped ancient Polynesian mariners navigate between the many islands of the Pacific Ocean.
C. Bright stars acting as compasses helped ancient Polynesian mariners navigate the many islands of the Pacific Ocean.
D. Bright stars acted as compasses, they helped ancient Polynesian mariners navigate between the many islands of the Pacific Ocean.

 

The original version contains a dangling modifier: the introductory phrase hanging low on the horizon modifies bright stars, not ancient Polynesian mariners (sailors) — logically, people cannot hang low on the horizon, as this answer implies.

(B) contains a fragment that lacks a main verb: acted “belongs” to which, rather to its subject, Bright stars.

(D) contains a comma splice: two complete sentences separated by only a comma.

(C) is correct because it eliminates the dangling modification and provides a main verb (helped) that accurately corresponds to its subject (Bright stars).

 


 

11/10/24

 

In 1977, the MIT professor Thomas J. Allen examined communication patterns among scientists and engineers and found that the farther apart their desks were, the less likely they were to communicate. At the 30-meter mark, the likelihood of regular communication approached zero. The expectation was that information technology would change that. Recently, therefore, researcher Ben Waber discovered that communication tools intended to erase distance are used largely among people who see one another face-to-face.

 

A. NO CHANGE
B. however,
C. moreover,
D. indeed,

 

The fact that the underlined transition is placed between two commas indicates that it serves to indicate the relationship between the sentence in which it appears and the previous sentence. In order to answer the question, you must consider the information in both sentences, and even before as well.

Sentence 1: people expected that information technology would change “that” (“that” = the fact that workers sitting far apart communicate infrequently).

Sentence 2: Ben Waber discovered that information technology is primarily used by workers sitting close to one another.

In other words, information technology did NOT actually change things: workers sitting far from each other still did not communicate. The sentences express contrasting ideas, so a contradictor is required. “However” is the only option that conveys a contrasting relationship, so (B) is correct. “Therefore,” “moreover,” and “indeed” are all used to convey similar ideas. (Note that however appears more frequently as an SAT answer choice than any other transition and that it is correct around 50% of the time.)

 


 

11/9/24

 

The planet Venus is believed to have contained Earth-like oceans at some point in they’re history, but these bodies of water evaporated as temperatures rose.

 

A. NO CHANGE
B. their
C. it’s
D. its

 

Logically, the pronoun in the blank should refer to the singular noun Venus — the sentence is discussing the history of that planet, not the history of oceans. A singular pronoun is thus required. (A) and (B) include forms of the plural pronoun they and can be eliminated.

(C) can be eliminated as well because you would not say ...Venus is believed to have contained Earth-like oceans at some point in it is history... That leaves (D), which correctly supplies the singular possessive, its.

 


 

11/8/24

 

It isn’t yet clear how much plastic is consumed by corals in the wild, or what harm it might do to these important marine organisms, which are already threatened by environmental dangers like warming seas and pollution. But understanding why plastic might appeal to them is important, especially because some particles appear to get stuck in the corals, potentially disrupting their digestive process.

 

Hundreds of chemicals are mixed into plastics to achieve certain textures or other characteristics. Because the corals sense the presence of food with receptors, it would not be all that surprising if some chemical additives mimicked substances that set off the corals’ appetites, suggested Alexander Seymour and Austin Allen, who were both graduate students at Duke University when they led this study.

 

In the last sentence, “set off” most nearly means

 

A. revealed
B. stimulated
C. responded
D. compensated for

 

The passage is discussing corals that consume plastics. Given that context, the “chemical additives” referred to in the last sentence must make the coral hungry, i.e., they must “stimulate” the corals’ appetites. “Compensated (paid) for” and “responded” do not make sense at all, and “revealed” does not quite fit: the passage indicates that additives cause corals to become hungry when they were not hungry before, whereas “revealed” would imply that the corals were already hungry.

 


 

11/7/24

 

Chunks of ice and dust, which make their home in corners of the galaxy far beyond Pluto, and sometimes become dislodged and enter the solar system as streaky comets.

 

A. NO CHANGE
B. Pluto, they
C. Pluto and
D. Pluto,

 

The key to answering this question is to recognize that the sentence contains a non-essential clause (which make their home in corners of the galaxy far beyond Pluto) that can be removed from the sentence. When the clause is removed the error becomes apparent: Chunks of ice and dust…and sometimes become dislodged and enter the solar system as streaky comets. When the word and is removed, a logical sentence is created. That makes (D) correct.

 


 

11/6/24

 

Thousands of years ago, two microscopic spores spawned and created a monster. It grew — up to three feet a year — sending out dark, gnarly, threadlike organs called rhizomorphs that explored the subterranean darkness, foraging for food. Now it’s a nebulous body, a tangled mat beneath the Oregon soil that occupies an area the size of three Central Parks and may weigh as much as 5,000 African elephants.

 

Its scientific name is Armillaria ostoyae, but you can call it The Humongous Fungus. It’s the largest known terrestrial organism on the planet, according to the United States Forest Service. It’s also a deadly forest pathogen.

 

Although none (that we know of) are as big, there are many others in the Armillaria genus. These fungi cause root rot disease in plants in forests, parks, orchards and vineyards across North America, Europe and Asia. What sets them apart from other fungi is those stringy rhizomorphs that find weak trees, colonize their roots, kill and eat them.

 

The passage indicates that “stringy rhizomorphs” are

 

A. beneficial to trees and other plants.
B. unique to members of the Armilliaria genus.
C. a common characteristic of fungi.
D. destroyed by exposure to light.

 

The answer is located in the last sentence. The key is to recognize that the word them in the phrase What sets them apart from other fungi is those stringy rhizomorphs… refers to fungi in the Armillaria genus. By definition, something that sets Armillaria fungi “apart” is unique to those fungi. That makes the answer (B).

(A) in incorrect because the passage indicates that Armillaria fungi are highly destructive to plants. (C) states exactly the opposite of what the passage indicates: string rhizomorphs are found only in the Armillaria genus; they are not a “common” characteristic of fungi. Be careful with (D): the passage only states that Armillaria ostoyae spreads in the dark — you cannot make a leap to infer that rhizomorphs are destroyed by light.

 


 

11/5/24

 

For years, seismologists have been trying to identify microquakes. Earthquakes so tiny they don’t even register on traditional measurement tools. Identifying microquakes can help scientists understand earthquake behavior and help them predict dangerous seismic events.

 

A. NO CHANGE
B. microquakes; earthquakes
C. microquakes, earthquakes
D. microquakes, and earthquakes

 

Shortcut: period = semicolon = comma + and. Since (A), (B), and (D) are grammatically identical, none of them can be correct. That leaves (C), which correctly uses a colon to introduce the definition of microquakes. Note that while a colon must be preceded by a complete, standalone sentence (For years, seismologists have been trying to identify microquakes), it can be followed by a fragment (Earthquakes so tiny they don’t even register on traditional measurement tools). In contrast, a period, semicolon, and comma + and can only be used to separate two complete sentences.

 


11/4/24

 

Self-portraiture isn’t just a byproduct of the smart phone. Since as early as the fifteenth century, artists across different mediums use self-portraits as a way to meditate on the world around them and their places within it. More than just capturing physical features, these images allow artists to channel their beliefs into their work in ways that are both revealing and revolutionary.

 

A. NO CHANGE
B. used
C. have used
D. had used

 

The word since, which appears at the beginning of the sentence, is a tip-off that the present perfect (have used) is required. This tense indicates that an action began in the past (fifteenth century) and is continuing into the present. Only (C) provides this tense, so it is correct.

 


 

11/3/24

 

The strings of letters that make up genes are largely useless on their own; instead, like blueprints for the many proteins in the body. To actually build something, or be expressed, certain genes must be switched on. Spaceflight seems to affect the level of this expression for some genes—especially those that play a role in the immune system, DNA repair, and bone growth.

 

A. NO CHANGE
B. own, instead like blueprints,
C. own. Instead, they are like blueprints
D. own instead being like blueprints

 

(A) is incorrect because a semicolon can only separate two sentences, and instead, like blueprints for the many proteins in the body is not a sentence. (B) is incorrect because the two commas signal a non-essential phrase; however, the sentence no longer makes sense when the information is removed (The strings of letters that make up genes are largely useless on their own…for the many proteins in the body). (D) is incorrect because some form of punctuation is required before instead to signal the start of a new clause. In addition, the word being reliably signals a wrong answer. (C) is correct because it adds a subject and verb (they are) to the second clause and uses a period to separate the resulting two sentences.

 


 

11/2/24

 

In some form or another, doughnuts have existed for so long that archaeologists keep turning up what look like fossilized bits of them in the middle of prehistoric settlements. But the doughnut proper, (if that’s the right word), supposedly came to Manhattan, then still New Amsterdam, under the unappetizing Dutch name of olykoeks—“oily cakes.”

 

A. NO CHANGE
B. proper, (if that’s the right word)
C. proper (if that’s the right word),
D. proper (if that’s the right word)

 

As a general rule, a non-essential clause can be set off by either commas or parentheses but not both. In rare cases, when a comma is necessary for other reasons, one may be placed after a close-parenthesis, but a comma before an open-parenthesis, or before and after a set of parentheses, is always incorrect. That automatically eliminates (A) and (B). Because there is no grammatical reason to place a comma after the close-parenthesis here, no commas are necessary, making the answer (D).

 


 

11/1/24

 

For cost-conscious clothing shoppers in 1920, it must have seemed like a miracle: men’s suits in a choice of 50 different styles for a mere 60 cents each (about $7.66 today). What’s more, when a suit got dirty, you could easily clean it—with an eraser. The first rubber erasers had been produced in England more than a century earlier. Paper clothing had arrived, largely imported from Germany and Austria, where World War I shortages of wool and other materials had spurred its development.

 

The writer is considering deleting the underlined information. Should the writer do this?

 

A. Yes, because the passage does not state that rubber erasers could be used to clean the suits.
B. Yes, because the passage focuses primarily on paper clothing.
C. No, because the passage indicates that paper clothing was erasable.
D. No, because the passage states that the suits seemed like a miracle.

 

“Delete” questions are essentially asking you one thing: whether the information in question is on- or off-topic. As a result, you must start by identifying the topic of the passage. What is the topic? Paper clothing (suits). What is the sentence in question about? The first rubber erasers? Is that the same thing as paper clothing? No, so the sentence should be deleted. Why? Because it’s off-topic (i.e., not about paper clothing). That makes the answer (B).

 


 

10/31/24

 

In the sixteenth century, Thomas More coined the term utopia to describe a perfect yet fictitious island society’s ways of life. Since then, people have tried to replicate this society, not just in stories but also in real life. Around the world, a handful of towns and cities have been designed with this ideal society in mind. Though inevitably they fall short of perfection, it’s still possible to visit some of them.

 

What is the best placement for the underlined word?

 

A. Where it is now.
B. Before the word fall.
C. Before the word perfection.
D. Before the word possible.

 

The fact that inevitably ends in –ly indicates that this word is an adverb, meaning that it can modify a verb (likely) or an adjective (less likely). Because modifiers must be placed next to the words/phrases they modify, it is reasonable to assume that inevitably will be placed next to a verb. The only answer to contain this construction is (B), which places inevitably next to the verb fall. It does make sense to say Though they inevitably fall short of perfection, so (B) is correct.

 


 

10/30/24

 

Twenty-five hundred miles from the nearest continental landmass and adrift in oceanic isolation, the Hawaiian islands are the most habitat-diverse place on the planet. Of its’ 1,200 native plant species, 90 percent exist nowhere else on earth.

 

A. NO CHANGE
B. its
C. there
D. their

 

Start by determining the noun to which the underlined pronoun refers — that is, of whose 1,200 native plant species? Logically, it must be the Hawaiian islands, plural. (A) can be eliminated because its’ does not exist, and (B) can also be crossed out because its is singular. Although there refers to a place, the correct version of the pronoun must be possessive because a noun phrase follows (1,200 native plant species) — and a noun can only possess another noun. Their is both plural and possessive, so (D) is correct. Their 1,200 native plant species = the 1,200 native plant species of the Hawaiian islands.

 


 

10/29/24

 

When fragments of China’s famed terracotta warriors were discovered by farmers in 1974, Zhao Kangmin was one of the first archaeologists on the scene. He consistently pieced the fragments together, spurring an excavation that would reveal thousands more clay soldiers packed into underground corridors.

 

Which choice most strongly emphasizes that Zhao Kangmin worked in a careful and detailed way?

 

A. NO CHANGE
B. compellingly
C. exclusively
D. meticulously

 

To work meticulously is to work “in a careful and detailed way,” so (D) is the correct answer. Note that you do not need to know the definition of this word to answer the question: as long as you can recognize that consistently, compellingly, and exclusively do not fit the required definition, you can use process of elimination to come up with (D).

 


 

10/28/24

 

The exhibition “Faces of Frida” includes not only paintings by Frida Kahlo and also her letters, personal photographs, and unpublished writings. Visitors can peruse the pages of her colorful diary, read her letters to her mother, Matilde Calderón y González, and browse through photos of Kahlo and her husband, the artist Diego Rivera.

 

A. NO CHANGE
B. along with
C. but also
D. in addition to

 

Not only, which appears in the non-underlined portion of the sentence, must be paired with but also. That makes (C) correct.

 


 

10/27/24

 

The race is on to build the world’s first meaningful quantum computer—one that can help scientists do things like develop miraculous new materials and encrypt data with near-perfect security. Although technology companies persistently tout each new breakthrough, next-generation systems will also need new algorithms, software, interconnects and a number of other yet-to-be-invented technologies.

 

Which choice most clearly expresses that the technology companies are excited about new breakthroughs?

 

A. NO CHANGE
B. reportedly
C. breathlessly
D. skeptically

 

To say that something is done breathlessly is to say that it was done excitedly — the implication is that an action or event was so thrilling that it takes someone’s breath away. Persistently and repeatedly imply that technology companies are forceful — but not necessarily excited — about publicizing each new breakthrough, and reportedly implies that the writer does not know for certain that the companies actually tout (proclaim) each new breakthrough. In contrast, breathlessly has a clear connotation of excitement. That makes the answer (C).


 

10/26/24

 

Experts use to think that nearly all nitrogen in soil came directly from the atmosphere, sequestered by microbes or dissolved in rain. But it turns out that scientists have been overlooking another major source of this element, which is crucial to plant growth: up to a quarter of the nitrogen in soil and plants seeps out of bedrock.

 

A. NO CHANGE
B. use to think,
C. used to think
D. used to thinking

 

Used to think is an expression that means “previously thought” — use to does not exist. That eliminates (A) and (B). (D) can be eliminated as well because used to thinking is un-idiomatic and creates a nonsense construction when plugged into the sentence. (C) provides the correct version of the idiom, making that answer right.

 


 

10/25/24

 

No tidal dweller captured Rachel Carson’s imagination like Ascophyllum nodosum, a rubbery dark green algae known widely on the Atlantic coast as rockweed. The biologist was most enchanted by rockweed’s double life—how its identity changed with the tides.

 

A. NO CHANGE
B. nodosum, a rubbery, dark green, algae
C. nodosum a rubbery, dark green algae,
D. nodosum; a rubbery, dark green algae

 

(B) can be eliminated easily because no comma should be placed between an adjective (green) and the noun it modifies (algae). The same is true for (D) because a semicolon can only be used to separate two complete sentences, and a rubbery, dark green algae known widely on the Atlantic coast as rockweed is not a sentence. (C) is incorrect because a comma must be placed after nodosum in order to separate the independent clause that begins the sentence (No tidal dweller captured Rachel Carson’s imagination like Ascophyllum nodosum) from the dependent clause that modifies it (a rubbery dark green algae known widely on the Atlantic coast as rockweed). (A) is correct because it includes the necessary comma after nodosum and eliminates the unnecessary comma before algae. Note that the comma after rubbery is a distraction here — the sentence can be correct either with it (because rubbery can be understood to modify dark green algae) or without it (because it is acceptable to say rubbery and dark green algae).

 


10/24/24

 

The tradition of using everyday objects in artwork is known as assemblage. London’s Tate Museum, which includes many assemblage-based works, traces its history back to Europe in the early 1900s when Pablo Picasso started making 3-D works with found objects. Therefore, some of the most famous twentieth-century assemblage artists, like the artist Robert Rauschenberg—born Milton Rauschenberg in Port Arthur, Texas—may have pulled their inspiration from work by African-American folk artists.

 

A. NO CHANGE
B. However,
C. Hence,
D. Moreover,

 

To answer this question, ignore the transition already in the passage, and consider the information before and after it. The previous sentence indicates that the Tate traces its history back to Europe in the early 1900s. The statement after the transition indicates that some of the most famous twentieth-century assemblage artists may have been inspired by African-American folk artists. Those are contrasting ideas, so a contradictor is required. Only however fits, making (B) correct.

 


 

10/23/24

 

More than 150 years ago, the first whales and dolphins were brought from the wild and into aquariums, and since that time, scientists learn an enormous amount about their intelligence and complex social lives.

 

A. NO CHANGE
B. learned
C. have learned
D. would have learned

 

The tip-off word for this question is since — it indicates that the sentence is describing an action that began in the past and that is continuing into the present (scientists began learning about dolphins’ intelligence and complex social lives 150 years ago and are still doing so today). That makes the answer (C).

 


 

10/22/24

 

Cheesemaking, which began 10,000 years ago, was originally about survival for a farm family or community: taking a very perishable protein (milk) and transforming it into something less perishable (cheese) so that there would be something to eat later. Many of us today think of cheese in the context of tradition, flavor, or family farms, but a basic goal—whether a producer is making farm-made cheddar or concocting the cheeseless industrially produced dairy product Velveeta, has always been getting as much edible food from a gallon of milk as possible.

 

A. NO CHANGE
B. Velveeta has
C. Velveeta—has
D. Velveeta has,

 

If you read the entire sentence in which the underlined portion appears, you’ll notice that it contains a dash after goal. That construction suggests that a second dash is needed later in the sentence, which is in fact the case. To confirm, cross out the information from whether to Velveeta, and read the sentence without it: Many of us today think of cheese in the context of tradition, flavor, or family farms, but a basic goal…has always been getting as much edible food from a gallon of milk as possible. The remaining sentence makes sense, so the information is not essential. When one dash is used to begin a non-essential clause, a second dash is required to end it. That makes (C) correct.

 


 

10/21/24

 

To evoke and conjure the spirit of the Rocky Mountains, the people of Stoney Nakoda have created artworks that have deep roots in the history of their homeland. A seemingly straightforward landscape image, for instance, pays homage to the region as a place to gather herbs, hold vision quests, and hunt.

 

A. NO CHANGE
B. To evoke while conjuring
C. To evoke yet conjure
D. To evoke

 

Evoke and conjure have the same essential meaning (“call forth” or “convey”), so it is only necessary to include one of these words. That makes (D) correct.

 


 

10/20/24

 

The Loch Ness Monster is one of Scotland’s oldest and most enduring myths, having inspired books, TV shows and films, and sustains a major tourism industry around its home. The story of the monster can be traced back 1500 years to 565 AD, when an Irish missionary is said to have encountered a beast in the River Ness. Later, in the 1930s, they announced the first modern sighting of the creature, dubbed “Nessie.”

 

A. NO CHANGE
B. one
C. we
D. reporters

 

Although the general meaning of the last sentence is understandable, it is also vague — there is no noun specifying who “they” are. As a result, a noun must be provided, making (D) the answer. Shortcut: given the choice between a noun and a pronoun, the noun will almost always be correct.

 


 

10/19/24

 

It’s hard to imagine now, but a little over a century ago, there was hardly anyone in the world whom knew what plastic was. Today, that material fuels modern life, from medical devices to the lightweight materials used in cars, computers, spaceships, and shopping bags.

 

A. NO CHANGE
B. who knew
C. which knew
D. in which they knew

 

(A) is incorrect because whom cannot be used before a verb. (C) is incorrect because which is used for things, not people, and anyone refers to people. (D) is a jumbled construction that creates nonsense when plugged in. (B) is correct because it uses who before a verb and refers to people.

 


 

10/18/24

 

Throughout the exhibitions at The American Jazz Museum, collections of photographs, sheet music, and posters from the height of jazz’s popularity creates context for historic artifacts such as Charlie Parker’s Grafton saxophone and a sequined gown worn by Ella Fitzgerald.

 

A. NO CHANGE
B. create
C. has created
D. creating

 

The answer choices include a mix of singular verbs (creates, has created) and plural verbs (create), so start by identifying the subject and determining whether it is singular or plural. In this case, you must look all the way back to the beginning of the clause, after the comma. The subject of the underlined verb is collections — all the information between that word and the verb belongs to the prepositional phrase of photographs, sheet music, and posters from the height of jazz’s popularity. Because collections is plural, the verb must be plural as well, making (B) correct.

 


 

10/17/24

 

It’s probably no surprise to dog owners, but growing research suggests that dogs often act more human than canine. They can: read facial expressions, communicate jealousy, display empathy, and even watch television.

 

A. NO CHANGE
B. can read facial expressions; communicate jealousy; display empathy, and even watch television.
C. can read facial expressions, communicate jealousy; display empathy; and even watch television.
D. can read facial expressions, communicate jealousy, display empathy, and even watch television.

 

(A) is incorrect because a colon must follow a complete, standalone sentence, but they can cannot stand on its own as a thought. To check (B), (C), and (D), look for the option that uses only commas or only semicolons — either can be used to separate the items in a list (although commas are almost always correct), but these two types of punctuation cannot be mixed and matched. (D) contains commas alone, so it is correct.

 


 

10/16/24

 

The Outer Space Treaty—written in 1967 and signed by all the major world powers—is the closest thing we have to a constitution for space. For a document conceived before the moon landing, it’s remarkably forward-looking: it declares “celestial bodies” like the moon and asteroids off-limits for private development and requires that countries authorize and continually supervise companies’ activities in space.

 

A. NO CHANGE
B. company’s activities
C. companies activity’s
D. companies activities’

 

When two-word apostrophe questions are tested, work on one word at a time. To check whether the first word requires an apostrophe—that is, whether it is possessive or plural—look at the second word. A possessive noun must be followed by another noun. Here, activities is a noun, so the first word requires an apostrophe. That eliminates (C). The second word is followed by in, which is not a noun, so no apostrophe should be used. That eliminates (D). To choose between (A) and (B), you must consider the passage. Company’s is singular, and it does not make sense to say that countries authorize and continually supervise a single company’s activities in space. Grammatically, this does not make sense either: a singular noun here would be preceded by a or the. The most logical interpretation of the passage is that countries must authorize and supervise the activities of multiple companies. Companies‘ is the plural possessive of company, making (A) correct.

 


 

10/15/24

 

Not only are parrots known for being a loud and destructive bird, but they are also highly intelligent and live up to 80 years. Nevertheless, the draw to keep them can be irresistible.

 

A. NO CHANGE
B. a loud, and destructive bird, and
C. loud and destructive birds, but
D. loud and destructive birds, and

 

(A) is incorrect because parrots are birds, not a single bird. (B) is wrong for the same reason, plus the fact that no comma should be placed between two adjectives separated by the word and (loud and destructive). (D) is incorrect because not only appears earlier in the sentence, and that phrase must be paired with but also. (C) contains but, so that answer is correct.

 


 

10/14/24

 

A few years ago, my family and I piled into our car and headed west for a summer to explore Yosemite National Park. I tasted the freedom of the open road and experienced the wonders of wild places. I had never seen such dramatic scenery before. Last August, therefore, I set out on an expanded version of that adventure, seeking to spend 10 months visiting all the national parks in the contiguous United States.

 

Which choice provides the most effective transition between the previous sentence and the information that follows?

 

A. NO CHANGE
B. I couldn’t wait to experience that kind of adventure again.
C. I didn’t want to come at first, but my family insisted.
D. I’d been camping before, but never for so long.

 

Although the question is phrased in terms of a “transition,” you must focus on the information that follows because the underlined sentence must set up that information — what comes before is less important. The sentence that follows indicates that the writer decided to visit all the national parks in the United States as a result of his/her experience in Yosemite, as indicated by the word therefore. As a result, the correct option must be positive and consistent with the idea of wanting to visit additional national parks. (C) and (D) are both off-topic, but be careful with (A): the statement that the writer had never seen such dramatic scenery before might seem to explain the desire to visit more parks, but (B) is much more specific: the phrase couldn’t wait to experience that kind of adventure again indicates a direct cause for the statement that follows. (B) is thus correct.

 


 

10/13/24

 

The idea for the first youth hostel was conceived in 1909 by Richard Shirrmann, an elementary school teacher living in the industrial center of Germany. Alarmed at the effects of the industrial revolution on his students’ health, he created a “wandering school” on weekends by taking students on field trips into the countryside for fresh air and exposure to nature. Students unrolled their bedding each night in school buildings, and the concept of student “youth hostels” was born.

 

A. NO CHANGE
B. effects of the industrial revolution,
C. affects of the industrial revolution
D. affects of the industrial revolution,

 

Affects is a verb, whereas effects is a noun. Here, the first word of the underlined portion follows the, so it must be a noun. That eliminates (C) and (D). (B) can be eliminated as well because when this answer is plugged in, a comma is incorrectly placed before a preposition (on). (A) does not contain the comma, so it is correct.

 


 

10/12/24

 

Since the invention of digital technology and audio sampling, films have been able to rely on digital samples to imitate the sound of live instruments. Today, many scores are created and performed wholly by the composers, themselves, through the use of sophisticated music composition software.

 

A. NO CHANGE
B. composers themselves,
C. composers, themselves
D. composers themselves

 

“Self” words should not be surrounded by commas; in most instances, it is also incorrect to place a comma only before or only after such words. Because no comma is required for a separate reason, no punctuation should be used. (D) is thus correct.

 


 

10/11/24

 

Wilhelm Von Osten firmly believed that humanity had greatly underestimated the reasoning skills and intelligence of animals. To test his hypothesis, he took it upon himself to tutor a cat, a horse, and a bear in mathematics. The cat was indifferent in his efforts, and the bear seemed outright hostile, but the Arab stallion named Hans showed some real promise.

 

A. NO CHANGE
B. to
C. at
D. from

 

The correct idiom is indifferent to; any other preposition is incorrect. That makes (B) the answer.

 


 

10/10/24

 

Cactuses are spiky and rough, foreboding and strange, gnarled and occasionally dangerous. They evoke harsh and unforgiving landscapes, but when they are removed from their native habitats, individually potted, and selling as decorations for a house, a garden, or an office, they are among the easiest plants, requiring little or no care and still looking good.

 

A. NO CHANGE
B. selling them
C. they sell
D. sold

 

The underlined portion is the third item in a list, and so it must be presented in the same format as the first two items. Both of those items contain past-tense forms of a verb (removed, potted), so the third item must contain that form as well. (D) is the only option that fits, making it the answer.

 

 


 

10/9/24

 

Color vision depends on our eyes and brain working together to perceive different properties of light. Although we see the natural and artificial light that illuminates our world as white, but it is actually a mixture of colors that, perceived on their own, would span the visual spectrum from deep blue to deep red. You can see this when rain separates sunlight into a rainbow or a glass prism separates white light into a multi-color band.

 

A. NO CHANGE
B. so
C. when
D. DELETE the underlined word.

 

As a rule, two consecutive clauses should not both begin with conjunctions. Because the first clause begins with a conjunction (although), the following clause cannot also begin with a conjunction (but). As a result, the underlined conjunction must be deleted, making the answer (D).

 


 

10/8/24

 

In 1984, several graffiti vandals were given the option of either going to jail or taking part in a new city beautification initiative. They chose the latter and became some of the first members of the Philadelphia Mural Arts Program. Since then, the program overseen the creation of more than 3,800 pieces of art painted on sides of buildings. 2,000 of the works are still viewable by the public, making this collection the “World’s Largest Outdoor Art Gallery.”

 

A. NO CHANGE
B. has oversaw
C. has overseen
D. oversaw

 

The word since at the beginning of the sentence is a tip-off that the present perfect (has + past participle) is required. (A) and (D) do not place has before the verb, so they can be eliminated. (A) is also incorrect because the simple past form of oversee is oversaw. (B) can be eliminated as well because has must be followed by the past participle, overseen. (C) is correct because it supplies the appropriate form.

 


 

10/7/24

 

One issue that often plagues even the most sturdily built old homes is the “improvements’’ made in the intervening decades. Whether the culprit was a clueless homeowner or an unlicensed contractor or an unskilled handyman, substandard repairs are common.

 

A. NO CHANGE
B. most sturdy built
C. more sturdier built
D. sturdier built

 

Built is a verb, and so it must be modified by an adverb — sturdily, not sturdy or sturdier. That eliminates (B) and (D). (C) is incorrect because it is redundant to use both more and the comparative form of an adjective (sturdier). (A) correctly uses the adverb sturdily to modify the verb built.

 


 

10/6/24

 

Tokyo is one of my favorite places, but it’s also one of the most expensive cities in the world. During my last visit I decided to participate in a homestay in order to save money on a hotel. Through a tourist website, I found Yuko, a young Japanese woman who rented out a spare room in her house. She seemed excited to hear from me and sent me useful information about public transportation, along with some maps. Tokyo was originally known as Edo, but its name was changed to Tokyo when it became the imperial capital in 1868. I could not wait to meet this interesting Japanese family.

 

Which choice provides the most relevant information at this point in the passage?

 

A. NO CHANGE
B. She also described her husband, who had a black belt in Karate, and her 85-year-old grandmother, who still rode a bicycle.
C. The house was located in Shinjuku, which is one of the busiest and most popular areas of the city.
D. Tokyo is divided into 23 districts, or wards, each of which is governed as an independent city.

 

To answer this question, you must focus on the information that comes after that underlined statement — the beginning of the passage does not provide enough information. If you read the sentence after the underlined section, you’ll learn that the writer could not wait to meet this interesting Japanese family. Logically, then, the previous sentence must provide specific information about how Yuko’s family was interesting. The only answer to provide such information is (B); (A) and (D) are entirely off-topic, and (C) is about the location of Yuko’s house, not about her family.

 


 

10/5/24

 

Born William Harrison Dempsey in Manassa, Colorado, “Jack” Dempsey was one of 11 children. He left home at the age of 16 and traveled west on freight trains with hobos, settling occasionally in mining towns. It was during that period of his life that Dempsey learned how to box. Initially, he only needed to defend himself at first, but soon he showed exceptional talent for the sport.

 

A. NO CHANGE
B. first of all
C. in the first place
D. DELETE the underlined portion.

 

To answer this question, you must consider the sentence as a whole; the underlined section alone does not give you enough information to determine the correct option. When considered independently, the underlined portion makes sense; however, the beginning of the sentence already contains the word initially, which is a synonym for at first. As a result, at first is redundant and should be eliminated, making the answer (D).

 


 

10/4/24

 

In 1931, RCA Victor developed and released the first 33 1/3 rpm records to the public. The format initially was a commercial failure because the records and playback equipment were expensive and because the audio performance was poor. When the stylus for playback was improved and the product released, the record became the standard in music playback for decades.

 

A. NO CHANGE
B. the product that was released,
C. the product being released,
D. the product has been released,

 

The presence of the word and before the underlined portion signals a parallel construction: the correct option must be parallel to the stylus for playback was improved. The expected answer would be the product was released, but that is not one of the options. The key to answering the question is to recognize that the word was is optional — the appearance of that verb before improved can “apply” to released as well, making the repetition unnecessary. As a result, (A) is correct. In (B), the word that creates a jumbled and un-parallel construction; in (C), being is awkward and again creates a construction that is not parallel; and in (D), has been is not parallel to was.

 


 

10/3/24

 

The moment a volunteer with a therapy dog walks into a hospital room, you can instantly feel a change in mood. Their eyes immediately focus on the animal, and smiles spread across their faces.

 

A. NO CHANGE
B. Your
C. One’s
D. Patients’

 

The reference to a hospital room implies that the underlined pronoun refers to patients, but because that noun does not actually appear earlier in the passage, it must be explicitly stated. Even if the meaning is clear enough, the sentence lacks precision. Only (D) provides the noun, making it correct. Shortcut: given the choice between a pronoun and a noun, the noun will usually be correct.

 


 

10/2/24

 

Throwing everything you own into your beat-up van and heading to California is a familiar path many aspiring musicians and actors follow as they head out West to chase their dreams. Although the odds are stacked quite heavily against them, opportunities are there for the taking. Such is the case for Richard Gibbs, a dreamer turned rock star and blockbuster film composer whose had a knack for being in the right place at the right time.

 

A. NO CHANGE
B. who’s had a knack for being
C. who’s had a knack to be
D. whose knack for being

 

Who’s = who is/has; whose = possessive of who. The possessive form must be followed by a noun, but the underlined word is followed by had, which is a verb. As a result, (A) and (D) can be eliminated. In (C), knack is followed by “to be” instead of by the idiomatically correct “for + -ing word” or “for + noun”. (B) is correct because it supplies the appropriate form of who and is idiomatically correct.

 


 

10/1/24

 

People are divided about whether hedgehogs should be kept as pets. Wild animals should stay wild, one argument goes. On the other hand, cats and dogs were wild once, and domestication has to start somewhere. At first, a hedgehog may feel threatened and extend they’re quills when handled by humans, but eventually most hedgehogs come to enjoy being petted and cuddled.

 

A. NO CHANGE
B. their
C. its
D. it’s

 

Start by identifying the noun to which the underlined pronoun most refers — what feels threatened and extends its quills? The most logical referent is a hedgehog, singular. They’re and their are both forms of they, plural, so (A) and (B) can be eliminated. (D) can also be eliminated because it’s = it is, and it does not make sense to say a hedgehog may feel threatened and extend it is quills. Its is the possessive form, making (C) correct. Its quills = the quills of a hedgehog.

 


 

9/30/24

 

During the 1988 games in Seoul, South Korea, Florence Griffith Joyner became an Olympic track and field champion that the phenomenon known as “Flo Jo” emerged. With her flashy one-legged running outfits, long hair, and brightly painted fingernails, she captured four medals and the world’s attention.

 

A. NO CHANGE
B. champion, and
C. champion, then
D. champion,

 

(A) is incorrect because that creates a jumbled sentence: this word must be followed by a verb, but here it is followed by a noun (the phenomenon). (C) and (D) are both incorrect because they create comma splices — two complete sentences separated by a comma. (B) is correct because comma + and = period, and a period can be plugged in to create two sentences: During the 1988 games in Seoul, South Korea, Florence Griffith Joyner became an Olympic track and field champion. The phenomenon known as “Flo Jo” emerged.

 


 

9/29/24

 

I still remember my first real scientific experiment—it was a high school microbiology exercise that involved growing bacteria in a petri dish. The experiment was very simple, but I’d never done anything remotely like it. When I checked on my results after a few days, I was shocked to see that the colonies have expanded.

 

A. NO CHANGE
B. are expanding.
C. had expanded.
D. would have expanded.

 

The passage is written in the past tense (was, involved, checked), so the underlined verb must be in the past tense as well. (A) does not work because have expanded is the present perfect — this tense is used to describe an action that began in the past and is continuing into the present. (B) does not work because are expanding is the present. (D) can be eliminated as well because would have is used to refer to actions that could have occurred but did not actually occur. That leaves (C): the past perfect (had expanded) is correctly used to indicate a past action (the colonies expanded) that came before another action (the writer was shocked).

 


 

9/28/24

 

Since ancient times, swans have been associated with tranquility and nobility, featuring in myths and stories around the world. Their high status is likely to have come about because of their perceived beauty and natural behavior: they are solitary birds, strong and aggressively protective of their young, but at the same time graceful and elegant on the water.

 

A. NO CHANGE
B. young—but
C. young but,
D. young

 

The fastest way to answer this question is to recognize that the comma after young in (A) signals the end of a non-essential clause. When that clause is crossed out, the sentence still makes sense: they are solitary birds…but at the same time graceful and elegant on the water. Because the non-essential clause begins with a comma, it must end with a comma as well. That eliminates (B). A comma cannot be placed after but, eliminating (C). And (D) can be eliminated because removing the transition creates a run-on when this answer is plugged in (they are solitary birds, strong and aggressively protective of their young at the same time graceful and elegant on the water). Again, that leaves (A).

 


 

9/27/24

 

Born in 1889, Ida Ten Eyck O’Keeffe was the third of seven children. She painted in both watercolors and oils: her grandmothers, Isabella Totto and Catherine O’Keeffe, were both painters, and she and her sisters Georgia and Anita were sent to study with a local artist, Sarah Mann, when they were young. While Anita did not choose to pursue an artistic career, another sister, Catherine, taught herself to paint and also became an artist.

 

Which choice provides the most effective transition between the beginning of the passage and information that follows?

 

A. NO CHANGE
B. Art ran in the family
C. During World War I, she worked as a nurse
D. She earned her master’s in fine arts from Columbia University

 

Although the question is phrased in terms of a “transition,” you must focus on the information that follows because the underlined sentence must set up that information — what comes before is less important. The portion of the sentence that follows the colon focuses on the fact that multiple members of O’Keeffe’s family participated in creating art. The only answer that information is consistent with is (B); all of the other options are off-topic.

 


 

9/26/24

 

In the eighteenth century, Linnaeus’s numerous books established a system of nomenclature that gave science an international language for identifying plants. Linnaean binomials, or two-word names, consisted of the genus (like a surname, designating a group of closely related plants) and the species (like a first name, designating the individual or specific kind of plant).

 

The writer is considering deleting the underlined portion (adjusting the punctuation accordingly). Should it be kept or deleted?

 

A. Kept, because it defines a term with which readers are unlikely to be familiar.
B. Kept, because it provides a specific example of a Linnaean binomial.
C. Deleted, because it shifts the focus from Linnaeus’s books to individual plants.
D. Deleted, because it does not indicate how plants were classified before Linnaeus.

 

The easiest way to approach this question is to spend a couple of seconds answering the question on your own, before you look at the choices. At the simplest level, the underlined information provides a definition — that is, it explains what Linnaean binomials are. With that information, you can quickly identify (A) as the most likely answer. And in fact, Linnaean binomials is a term most readers will probably not know, so the underlined information should be retained. All of the other answers are off-topic.

 


 

9/25/24

 

The phrase “barren sand flat” does not typically inspire creativity. It is this condition, however, that makes the bottom of the Gulf of Mexico the perfect spot for the United States’ first Underwater Museum of Art (UMA). Like similar undertakings near Cancun, the museum is best suited for scuba divers. On clear days, snorkelers can enjoy the submerged journey which showcases seven sculptures at a depth of around 60 feet and intermingles responsible tourism, environmentalism, and creativity.

 

A. NO CHANGE
B. journey, which showcases seven sculptures
C. journey which showcases seven sculptures,
D. journey which showcases: seven sculptures

 

The easiest way to answer this question is to know that which must follow a comma. That makes (B) the only possible answer. Otherwise, you can eliminate (C) because the comma after sculptures incorrectly places a comma before a preposition (at) when this answer is plugged into the sentence, and you can eliminate (D) because a complete, standalone sentence must come before a colon. To decide between (A) and (B), however, you must know the rule for which and commas.

 


 

9/24/24

 

Gaze-following is instinctual for many animals—including chimpanzees, goats, dolphins, and even the red-footed tortoise—because it alerts them to everything from imminent threats to potential sources of food.

 

A. NO CHANGE
B. imminent threats,
C. eminent threats
D. eminent threats,

 

Imminent means “about to occur,” whereas eminent means “distinguished” or “well-known.” Only the first definition makes sense in context of the passage, eliminating (C) and (D). (B) is also incorrect because this answer places a comma before a preposition (to) when it is plugged into the sentence; (A) correctly omits the comma.

 


 

9/23/24

 

Founded by art historian and collector William Arnett, Souls Grown Deep traces the history of many Outsider artistic creations back to the collapse of the agricultural economy in the aftermath of the Civil War, when African Americans were forced to migrate from rural areas to larger cities in search of work. During that period, folk artists had practical as well as stylistic reasons to use scavenged materials in that period: many of them were poor, so they worked with what they had.

 

A. NO CHANGE
B. throughout that period
C. during this period
D. DELETE the underlined portion.

 

The sentence already begins with the phrase During that period, so the inclusion of that information at the end of the sentence as well is redundant. As a result, it should be deleted, making (D) correct.

 


 

9/22/24

 

Although seaweed harvesting is hardly a new industry—New England’s farmers have nourished their fields with “sea manure” for centuries—rockweed has lately become a valuable commercial product. An ingredient in everything from fertilizers to pet foods to nutritional supplements.

 

A. NO CHANGE
B. product, an ingredient
C. product; an ingredient
D. product, it is an ingredient

 

You can identify the answer to this question with near-certainty using shortcuts alone: a period and a semicolon are grammatically equivalent, so (A) and (C) can be conclusively eliminated—if two answers are equivalent, neither can be correct because a question cannot have two right answers. In this case, these options are wrong because an ingredient in everything from fertilizers to pet foods to nutritional supplements is not a sentence. In (D), the construction comma + it suggests that this answer contains a comma splice—two complete sentences separated by a comma. And in fact, that is the case. That leaves (B) as the answer.

 


 

9/21/24

 

In the early 1900s, photographer David Fairchild trained his camera on a part of the world most of us ignore: the insects under our feet. His resulting body of work, published in 1913 in National Geographic magazine, was unique not only for its subject matter but also to its use of magnified images that showed bugs in intricate detail.

 

A. NO CHANGE
B. with
C. for
D. at

 

The presence of the word pair not onlybut also indicates that the preposition on each side of the word pair must be the same. Not only is followed by for, so but also must be followed by for as well. That makes the answer (C).

 


 

9/20/24

 

In the mid-nineteenth century, almost everyone in the United States agreed that women’s clothing posed a problem. The dictates of modesty called for floor-length dresses, and fashion demanded a full skirt beneath a tiny waist. Some women squeezed themselves into corsets and six to eight petticoats to fill out the shape of their skirts. The result weighed up to 15 pounds, placed enormous pressure on their hips, and movement was a struggle.

 

A. NO CHANGE
B. made movement a struggle.
C. they struggled to move.
D. a struggle for movement occurred.

 

The underlined portion involves the third item in a list, so the format of this item must match the format of the previous two. Each of those items begins with a verb (weighed, placed), so the third item must begin with a verb as well. Only (B) contains the correct construction (made), so it is correct.

 


 

9/19/24

 

Working from repurposed natural, industrial, and mass-produced objects, artist Gabriel Kuri explores the potential for transformation by looking at familiar things from unexpected angles. Everyday objects become a part of his sculptures, plastic bags, advertising flyers, and receipts.

 

A. NO CHANGE
B. Everyday objects, including plastics bags, advertising flyers, and receipts, become part of his sculptures.
C. Plastic bags, advertising flyers, and receipts become part of his sculptures, being everyday objects.
D. Part of his sculptures, plastic bags, advertising flyers, and receipts, which are everyday objects.

 

As a rule, modifiers must be placed as close as possible to the words/phrases they modify. The list plastic bags, advertising flyers, and receipts must refer to everyday objects, and so the two phrases must be placed next to one another. (A) creates a misplaced modifier by placing the list next to sculptures and can thus be eliminated. (C) is incorrect because these two elements are again separated, and because the phrase being everyday objects contains an unnecessary gerund (being). (D) is incorrect because it contains a fragment — the verb are “belongs” to the subject which rather than to its logical subject: plastic bags, advertising flyers, and receipts. As a result, the sentence is missing a main verb. (B) is correct because the word including makes it clear that the list is providing examples of everyday objects.

 


 

9/18/24

 

Imagine standing in front of a wall of windows, surveying the view. You hear someone enter the room behind you. You turn. “Welcome,” you say. “Here is the video, I wanted to show you.” At the press of a button, the view vanishes, and the windows transform into a high-definition television screen.

 

A. NO CHANGE
B. video I wanted
C. video, that I wanted
D. video that, I wanted

 

The underlined phrase can be written two ways: Here is the video I wanted to show you, or Here is the video that I wanted to show you. As a general rule, no comma should be used before or after that, eliminating (C) and (D). When that is not used, no comma should be used in its place, eliminating (A). (B) correctly omits the unnecessary punctuation.

 


 

9/17/24

 

From its very first moments in print on March 20, 1852, Harriet Beecher Stowe’s Uncle Tom’s Cabin was an enormous success. It sold 3,000 copies on its first day, and Frederick Douglass reported that 5,000 copies—the entire first print run—were purchased within four days.

 

Which of the following would NOT be an acceptable alternative to the underlined portion?

 

A. a rousing
B. a smashing
C. a burning
D. an overwhelming

 

It is idiomatically acceptable to say that something was a rousing, smashing, or overwhelming success. Only burning cannot be used to indicate a high level of success. Because the question asks you to identify the answer that is NOT acceptable, (C) is correct.

 


 

9/16/24

 

The myth of the disappearing book isn’t new. As early as 1894, there was speculation that the introduction of the phonograph would spell the demise of print books, which would be replaced by what we today call audiobooks.

 

A. NO CHANGE
B. label
C. annotate
D. decipher

 

To spell the demise of something is an idiomatic phrase meaning “to signal the downfall.” None of the other options have this meaning, making (A) correct.

 


 

9/15/24

 

I have always been intrigued by Thanksgiving—the traditions, the meal, the idea of a holiday that is simply about being thankful. For my family, Thanksgiving is all about the food, some of it is typical, but there are a few twists. The reliability of the menu—with a little flexibility sprinkled in—seems to unite us as a family while acknowledging our different backgrounds.

 

A. NO CHANGE
B. food. Some of it is typical,
C. food, and some of which is typical,
D. food; some typical,

 

This question is essentially testing the distinction between pronoun + of them (subject, can be used to begin an independent clause) and pronoun + of which (not a subject, can only begin a dependent clause). (A) can be eliminated because the comma before some of it creates a comma splice—two complete sentences separated by a comma. (C) is incorrect because comma + and = period, but a period cannot be plugged in without creating a fragment: For my family, Thanksgiving is all about the food. Some of which is typical, but there are a few twists. (D) is incorrect because a semicolon can only be used to separate two sentences, and some typical, but there are a few twists is not a sentence. That leaves (B), which correctly places a period between two complete sentences.

 


 

9/14/24

 

Imagine you’re a cat, and every time you meow, the loud voice of a snooty-sounding British gentleman kindly conveys your every thought and feeling to your human guardian. A product called the Catterbox—the world’s first talking cat collar—believes to do just that. A microphone and a speaker are used to capture a cat’s meow and translate it into an English-speaking human voice.

 

A. NO CHANGE
B. purports
C. insists
D. allows

 

Based on the context, the underlined word must mean something like “claims.” Believes, insists, and allows neither match that definition nor fit grammatically. Don’t get sidetracked by (D): you can say x allows someone to do y, but you cannot say x allows to do. (B) is the only option that fits: purports means “makes a claim to.” Note that even if you do not know the definition of this word, you can still answer the question through process of elimination.

 


 

9/13/24

 

Sneakers as we know them might never of came to be without Charles Goodyear’s invention of vulcanized rubber. Despite his poor business sense, the inventor saw an opportunity for technological advancement when he purchased hundreds of rubber life preservers that had melted in the summer heat. After years of experimentation, Goodyear finally happened upon the combination of lead, sulfur, and heat that allowed the rubber to keep its shape.

 

A. NO CHANGE
B. of come to be
C. have came to be
D. have come to be

 

Could, would, should, and might can only be followed by have, not of, eliminating (A) and (B). In addition, the past participle (come) rather than the simple past (came) must follow any form of the verb to have. That eliminates (C) and makes (D) correct.

 


9/12/24

 

Physicists trace the history of quantum theory back to 1927, when German physicist Werner Heisenberg showed that the classical physics methods did not apply to very small objects. When someone throws a ball, for instance, it’s easy to determine exactly where the ball is, and how fast it’s moving. But as Heisenberg showed, that’s not true for an atom or a subatomic particle. Instead, an observer can see either where it is and how fast it’s moving – but not both at the exact same time.

 

A. NO CHANGE
B. and if it’s moving fast
C. or how fast it’s moving
D. or is it moving fast

 

Right before the underlined portion, the word either appears (an observer can see either where it is…). Either must be paired with or, so (A) and (B) can be eliminated. As a rule, the constructions on either side of a word pair must match, so the underlined portion must be parallel to where it is. Only (C) contains the same general construction (relative pronoun + subject + verb), making it the answer.

 


 

9/11/24

 

The stamp celebrating the launch of the U.S. airmail service was an impressive sight. It featured a Curtiss JN-4 or “Jenny”, the same plane set to deliver the mail, and was printed in carmine rose and deep blue. The striking color scheme no doubt wooed buyers, but it also signaled a printing error. It was just the second time the Postal Service had attempted a two-color stamp, and with the fervor of World War I, sloppy mistakes were a more likely occurrence.

 

Which of the following would NOT be an acceptable alternative to the underlined word?

 

A. pursued
B. enticed
C. charmed
D. pleased

 

Note that it does not matter whether you know what wooed or enticed means — you do not need to know the definition of the original word in the passage, and if you can identify the right (i.e., wrong) answer, the meanings of the other answers become irrelevant. In this case, pursued simply does not make sense: a feature of a product (the striking color scheme) cannot chase after, i.e., pursue, buyers. Because the question asks you to identify the answer that is NOT acceptable, (A) is correct. Enticed, charmed, and pleased are all acceptable because they indicate that the striking color scheme was attractive or appealing to buyers.

 


 

9/10/24

 

The Kingikmiut Nanuuq Patrol—or the Wales polar bear patrol—resulted from an innovative partnership between the tribal council in Wales, Alaska United States government wildlife officials, and the World Wildlife Fund (WWF). Patrollers are trained to chase visiting polar bears out of town using an escalating range of deterrents. The goal is simple: to keep people safe from bears, while also keeping bears safe from people.

 

A. NO CHANGE
B. Wales, Alaska; United States government wildlife officials, and the World Wildlife Fund (WWF).
C. Wales, Alaska, United States government wildlife officials; and the World Wildlife Fund (WWF).
D. Wales, Alaska; United States government wildlife officials; and the World Wildlife Fund (WWF).

 

When questions test punctuation between the items in a list, the correct answer will almost always include commas rather than semicolons. However, semicolons are also acceptable. The only inviolable rule is that the punctuation must remain consistent—commas and semicolons may not both be used. In this case, the only answer that uses the same type of punctuation between all the items is (D), which happens to use semicolons. As a result, that answer is correct.

 


 

9/9/24

 

On June 30, 1908, an explosion ripped through the air above a remote forest in Siberia, near the Tunguska River. It destroyed 2,000 square kilometers of forest, flattening about 80 million trees. The earth trembled. Windows were shattered in the nearest town, more than 35 miles away. Residents even felt heat from the blast, and some were blown off their feet.

 

Which choice provides the most relevant detail at this point in the paragraph?

 

A. NO CHANGE
B. Researchers are unsure whether the explosion was caused by an asteroid or a meteoroid.
C. Over the last century, over 1,000 papers have been published about the explosion.
D. An explosion of this magnitude would have destroyed a larger metropolitan area.

 

To answer this question, focus on the statement that comes after the underlined portion — the preceding sentences do not provide enough information. The last sentence refers to residents who felt heat from the blast, so logically, the correct answer must give some idea of who those “residents” were. (B) and (C) are completely off-topic and can be eliminated, but be careful with (D): this answer is describing a hypothetical situation (what would have happened in a larger metropolitan area), whereas (A) refers to a specific place (the nearest town) and clearly indicates where the residents lived. (A) is the stronger answer and is thus correct.

 


 

9/8/24

 

For most of our lives, we use language to categorize objects, colors, emotions, and pretty much everything meaningful. Although our eyes can perceive thousands of colors, the way we communicate about them – and the way we use color in our everyday lives – requires us to carve this huge variety up into identifiable, meaningful categories. Painters and fashion experts, for example, use color terminology to refer to and discriminate between hues and shades that non-experts typically describe with one simple term.

 

A. NO CHANGE
B. nevertheless,
C. consequently,
D. meanwhile,

 

The underlined transition appears between two commas, indicating that it is being used to signal the relationship between the sentence in which it appears and the sentence(s) before — it does not connect two parts of the same sentence. As a result, you must start by considering the relationship between the information preceding the last sentence and the last sentence. The previous sentence indicates that people must categorize colors in order to communicate meaningfully about them. The final sentence then illustrates this idea by citing painters and fashion experts who must make fine distinctions between colors for professional reasons. The last sentence is therefore used as an example, making (A) correct.

 


 

9/7/24

 

At the age of 51, Georgia O’Keeffe was asked by the Philadelphia advertising agency N.W. Ayer & Son to travel to Hawaii to produce two print-ad images for the Hawaiian Pineapple Company, later renamed Dole. Not known for commercial work, O’Keeffe had completed a commission in 1936 — what would be the largest of her flower paintings — for the Elizabeth Arden Sport Salon in New York.

 

A. NO CHANGE
B. agency N.W. Ayer & Son,
C. agency, N.W. Ayer & Son,
D. agency, N.W. Ayer & Son

 

When names in the middle of a sentence are tested, there are typically only two correct options: two commas (non-essential) and no commas (essential). To check which one is correct, cross out the name, and read the sentence without it. At the age of 51, Georgia O’Keeffe was asked by the Philadelphia advertising agency…to travel to Hawaii to produce two print-ad images for the Hawaiian Pineapple Company, later renamed Dole. Although the sentence still makes grammatical sense, it does not really make sense in terms of meaning — we do not know what the Philadelphia advertising agency was called. As a result, the information is essential, and no commas should be used. That makes (A) correct.

 


 

9/6/24

 

Frank J. Sulloway, a psychology professor at the University of California, Berkeley believes that family roles based on birth order and competition between siblings affect a person’s behavior and eventually shape your personality.

 

A. NO CHANGE
B. one’s
C. their
D. his or her

 

The underlined pronoun logically refers to a person, singular, so the correct answer must be singular. That eliminates (C). (A) and (B) can also be eliminated because one’s should be paired with one, and your should be paired with you. His or her is singular and correctly corresponds to a person, making (D) the answer.

 


 

9/5/24

 

José Martí considered New York his adopted hometown and wrote dazzling accounts of the city, likening the cables of the brand new Brooklyn Bridge, to satisfied colossal boa constrictors resting on top of towers. He also rarely passed up an opportunity to let the city amuse him and was an enthusiastic patron of H.M. Barnum’s circus, billed as “The Greatest Show on Earth.”

 

A. NO CHANGE
B. Bridge to satisfied colossal boa constrictors resting,
C. Bridge to satisfied colossal boa constrictors resting
D. Bridge to satisfied, colossal, boa constrictors resting

 

(A) is incorrect because it places a comma before a preposition (to). When it is plugged in, (B) creates the same error: the comma after resting is placed before the preposition on. (D) is incorrect because it places a comma between an adjective (colossal) and the noun it modifies (boa constrictors). (C) is correct because no punctuation is necessary in the underlined section.

 


 

9/4/24

 

When Arcosanti opened in 1970, its founder, the Italian architect Paolo Soleri, imagined the small Arizona desert complex would of became a city of thousands of people, all living together in harmony in what he called an arcology—a community where nature and architecture work together to create a balanced existence.

 

A. NO CHANGE
B. would have became
C. would become
D. became

 

Would can only be followed by have, not of, eliminating (A). (B) can be eliminated because any form of to have must be followed by the past participle (become) rather than the simple past (became). Although (D) is in the simple past and is thus parallel to the other verbs in the passage, it does not make logical sense: Soleri could not have imagined that Arcosanti became something right when it opened. Logically, this verb must describe Soleri’s hopes for Arcosanti’s future. When the future is discussed from the perspective of the past, would + verb must be used, making the answer (C).

 


 

9/3/24

 

In the 1930s, the concept of showing movies outdoors wasn’t new: people often watched silent films on screens set up at beaches or other places boasting an abundance of sky. However, it was an auto-parts salesman named Richard Hollingshead whom saw the genius in giving a car-loving society one more activity to do in their vehicles.

 

A. NO CHANGE
B. who
C. which
D. and

 

As a rule, who rather than whom must be used before a verb, eliminating (A). (C) is incorrect because the underlined pronoun refers to a person, and which can only refer to things. (D) is incorrect as well because plugging in and creates a nonsense construction (However, it was an auto-parts salesman named Richard Hollingshead and saw…). (B) correctly uses who to refer back to Richard Hollingshead.

 


 

9/2/24

 

On a gloomy Wednesday morning, thousands of spectators gathered in Washington, D.C.’s Potomac Park to witness what would be the world’s first regularly scheduled airmail service. As the crowd buzzed with excitement, President Woodrow Wilson stood with the pilot, Second Lieutenant George Leroy Boyle. The two men chatted for a few minutes, Wilson in a three-piece suit and bowler hat, Boyle in his leather flying cap, a cigarette in his mouth. The president dropped a letter in Boyle’s sack, and the pilot took off for his journey from Washington, D.C., to New York, with plans to stop in Philadelphia for delivery and refueling. The flight, however, never made it to the City of Brotherly Love.

 

The writer is considering deleting the underlined portion of the sentence (replacing the comma after minutes with a period). Should the writer do this?

 

A. Yes, because it is irrelevant to the paragraph’s focus on the airmail service’s first flight.
B. Yes, because it suggests that Wilson and Boyle disagreed about the importance of the airmail service.
C. No, because it provides a detailed description that helps the reader envision the encounter between Wilson and Boyle.
D. No, because it emphasizes that Wilson and Boyle were able to overcome their differences.

 

Remember that “delete” questions are, in fact, asking you to determine two things: first, what type of information does the underlined portion contain, and second, is that information relevant? In this case, the underlined portion provides a description of Wilson’s and Boyle’s appearances. That information alone points to (C), and the explanation that answer provides makes sense as well: those details do, in fact, help the reader to envision the scene, and so they should NOT be removed. All of the other answers are unrelated to the idea of a detailed description, making (C) correct.

 


 

9/1/24

 

In southern Manitoba, a bison kept escaping a ranch. The locals nicknamed him Freddy, and he became something of a minor celebrity. He even had a song dedicated to him. After witnessing the media coverage surrounding Freddy, a renaissance-style choral arrangement was inspired to be crafted by composer Elliot Britton, complete with contemporary pop-chord progressions accompanied by traditional fiddle and electronically distorted bison noises.

 

A. NO CHANGE
B. the crafting of a renaissance-style choral arrangement was inspired by composer Elliot Britton.
C. composer Elliot Britton was inspired to craft a renaissance-style choral arrangement,
D. composer Elliot Britton, who was inspired to craft a renaissance-style choral arrangement,

 

Who witness[ed] the media coverage surrounding Freddy? Composer Elliot Britton. So composer Elliot Britton, the subject, must be placed at the beginning of the underlined phrase; otherwise, a dangling modifier is created. That eliminates (A) and (B). (D) can also be eliminated because this option creates a fragment: the verb was “belongs” to who rather than to composer Elliot Britton, and so the sentence lacks a main verb. The error is absent from (C), making it correct.

 


 

8/31/24

 

Researchers at the University at Buffalo have developed a solar water purifier they hope can sanitize water more quickly, cheaply, and effectively than other models. The device resembles a small A-frame tent. Black carbon-dipped paper is draped over a triangular form and set on top of the water. The edges of the paper trail in the water, soaking it up like a sponge.

 

What is the best way to combine the underlined sentences?

 

A. A triangular form is set on top of the water and draped with carbon-dipped paper, whose edges trail in the water, soaking it up like a sponge.
B. A triangular form is set on top of the water, and carbon-dipped paper is draped over it, whose edges trail in the water and soak it up like a sponge.
C. Black carbon-dipped paper is draped over a triangular form and, then, set on top of the water, its edges trailing and soaking it up like a sponge.
D. Draped over a triangular form, black carbon-dipped paper is set on top of the water, whose edges trail, soaking it up like a sponge.

 

(B) is incorrect because whose edges trail in the water and soak it up like a sponge modifies carbon-dipped paper and should be placed next to it; however, the modifying phrase is placed after the word it (that is, water), creating a misplaced modifier. (C) is incorrect because its edges trailing and soaking it up like a sponge is used to modify water rather than carbon-dipped paper, creating the same error as in (B). (D) is likewise wrong because the construction water, whose edges trail is illogical because water does not have edges. Rather, it is the edges of the carbon-dipped paper that trail in the water. (A) is correct because the phrase whose edges trail in the water is clearly used to modify carbon-dipped paper.

 


 

8/30/24

 

In the late nineteenth century, the most sophisticated railroad managers and some economists argued that railroads were “natural monopolies,” the inevitable consequence of an industry that required huge investments in land and construction. However, competition was expensive and wasteful. In 1886, the Atchison, Topeka, and Santa Fe Railway and the Missouri Pacific Railroad both built railroad tracks heading west from the Great Bend of the Arkansas River in Kansas to Greeley County on the western border, roughly 200 miles away. The tracks ran parallel to each other, about two miles apart.

 

A. NO CHANGE
B. Therefore,
C. Indeed,
D. Still,

 

To answer this question, ignore the transition already in the passage, and consider the information before and after it. The previous sentence describes the idea that the railroads were natural monopolies because they required huge investments in land and construction, and the sentence begun by the transition states that competition [between railroad companies] was expensive and wasteful. Those are similar ideas, so (A) and (D) can be eliminated (still means “despite this”). (B) is incorrect as well because the fact that competition cost a lot of money and created waste was not a result of the fact that it required huge investments. Those ideas are connected, but the first is not the cause of the second. (C) is correct because indeed is used to emphasize a preceding statement, which is exactly what the rest of the sentence does. The example that follows then further supports that idea by illustrating how competition led to wasteful spending (that is, two sets of tracks built almost right next to one another).

 


 

8/29/24

 

In the early twentieth century, new knowledge about nutrition science fueled widespread “expert” condemnation of dishes featuring a range of ingredients mixed together. Instead, reformers insisted with great confidence (but scant evidence), that it was healthier to eat simple foods with few ingredients—meals in which meats and plain vegetables were clearly separated.

 

A. NO CHANGE
B. confidence, (but scant evidence),
C. confidence, (but scant evidence)
D. confidence (but scant evidence)

 

As a general rule, it is unnecessary for parentheses to be accompanied by commas, either before or after. Because two commas and two parentheses both serve to indicate non-essential information, it is redundant to include both. (In rare cases, a comma may be required after a close-parenthesis for other reasons, but that is not relevant to this question.) (D) is the only option that does not contain commas, so it is correct.

 


 

8/28/24

 

Debbie Smith has her work cut out for her. Since 2010, she has been the artist responsible to record the likeness of every clown registered with Clowns International, the oldest established organization for clowns in the United Kingdom. It’s a seemingly straightforward task—that is, until you discover what she uses as a canvas: eggs.

 

A. NO CHANGE
B. for recording
C. in recording
D. with recording

 

The correct idiom is responsible for + -ING. The infinitive (to record) or a preposition other than for cannot be used. As a result, (B) is correct.

 


 

8/27/24

 

Bill Bowerman’s “eureka” moment came while eating breakfast with his wife on a summer Sunday in 1976. As he stared at his waffles, it occurred to him that the grooves of the waffle iron were a perfect mold for multi-terrain sneaker soles. He poured molten rubber into iron after iron that he perfected the waffle-sole pattern that Nike, which he co-founded in 1964, continues to use on some running and training shoes today.

 

A. NO CHANGE
B. which
C. until
D. when

 

If the original sentence doesn’t make sense to you when you read it, try to avoid falling into a loop of continually reading and re-reading it; when you can’t figure out what a sentence is trying to say, that’s a pretty reliable sign that something is wrong with it. In this case, once you’ve determined that that doesn’t make sense, you are probably best served by ignoring it and plugging in each of the answers in turn. The only option that makes sense is (C) — logically, Bowerman must have poured molten rubber into iron after iron until he perfected the waffle-sole pattern. The other answers all create nonsense meanings.

 


 

8/26/24

 

During the Renaissance, first-person accounts of little-explored lands and botanical discoveries thrilled armchair gardeners, working horticulturists and scholars, although the high cost of producing books and manuscripts tended to limit their audience. At a more practical level, interest in garden design and new techniques of cultivation blossomed and was accompanied by a combustion of interest in previously unknown plants.

 

A. NO CHANGE
B. a bang
C. a blowup
D. an explosion

 

To say that there was an explosion of interest in something is to say that there was a dramatic increase in interest. Although the other answers have similar literal meanings to explosion, none of them can be used idiomatically to have this meaning. (D) is thus correct.

 


 

8/25/24

 

In 2004, Debra Britt and her sisters, Felicia Walker and Tamara Mattison, began to collect and make dolls, doll clothes, and accessories. By 2012, the serious hobby had overrun their home, so they rented a storefront space in downtown Mansfield, Massachusetts, where they were living, and transformed it into the National Black Doll Museum of History and Culture.

 

A. NO CHANGE
B. have lived
C. would live
D. would have lived

 

If you look at the rest of the passage, you can see that the other verbs are in the past tense (began, rented, transformed), so the underlined verb must be in the past tense as well. (B) can be eliminated because have lived is in the present perfect, which describes an action that began in the past but that is continuing into the present. (C) and (D) can also be eliminated because would live/would have lived are both used to indicate hypothetical actions — actions that could take place, or could have taken place, that did not actually occur. That leaves (A): although were living is not precisely parallel in form to the other verbs in the sentence, it is still in the past and is therefore sufficiently parallel to be correct. The progressive (were…-ING) is simply used here to emphasize that the sisters’ residence in Mansfield was an ongoing situation.

 


 

8/24/24

 

For most of history, humans weren’t interested in the direct consumption of milk. Instead, the early milkers of the fertile crescent transformed it into sour yogurt, butter and cheese because the hot climate caused milk to quickly spoil. Even so, milk was a vital symbol in the mythology of the Sumerians, Greeks and Egyptians.

 

Which of the following is the LEAST acceptable placement for the underlined word?

 

A. where it is now.
B. after the word climate.
C. after the word milk.
D. after the word spoil.

 

This is a question that needs to be done more or less by ear. (A) is incorrect (i.e., acceptable) because quickly (adverb) can be used to modify spoil (verb). (B) is acceptable because quickly can modify the verb caused. (D) is acceptable because it is equivalent to (A) — it merely places the adverb after the verb, rather than before it. (C) is the least acceptable option because adverbs typically are not placed before infinitives (to spoil); the construction is not idiomatic. That makes (C) correct.

 


 

8/23/24

 

In James Dinh’s proposal for the National Museum of the American Indian’s new memorial, concentric circles—“ripples,” in Dinh’s imagination—radiate outward from a star and fountain and is bounded on one side by a mound of earth inlaid with a stone wall. One stretch of this wall, which Dinh terms the “Wall of Stories,” is particularly striking: it features a seated bronze sculpture of a mother and child.

 

A. NO CHANGE
B. are
C. has been
D. have been

 

The answer choices contain both singular and plural verbs, indicating that this question is testing (in part) subject-verb agreement. To make the subject easier to identify, ignore the non-essential clause between the dashes: In James Dinh’s proposal for the National Museum of the American Indian’s new memorial, concentric circles…radiate outward from a star and fountain and is bounded on one side by a mound of earth inlaid with a stone wall. The subject is concentric circles (plural), so a plural verb (are) is required. That eliminates (A) and (C). Next, determine the tense: all of the other verbs in the passage are in the present (radiate, terms, is, features), so the underlined verb must be in the present as well. That makes the answer (B).

 


 

8/22/24

 

In a 2012 paper, marketing researchers Rajeev Batra, Aaron Ahuvia and Richard P. Bagozzi developed a model of “brand love.” Based on studies of consumers’ brand attachment, they showed that in order to form meaningful attachment with a brand, consumers need to experience them in ways that go beyond simply buying and using a product.

 

A. NO CHANGE
B. themselves
C. itself
D. it

 

The only plural noun to which the underlined pronoun could refer is consumers, but it does not make sense to say consumers need to experience consumers in ways that go beyond simply buying and using a product. A far more logical interpretation of the sentence is that the antecedent is the singular noun a brand. As a result, a singular pronoun must be used. That eliminates (A) and (B). (C) is incorrect because “self” words are used to indicate that someone/something is both the subject and the object of an action, and a brand cannot do anything to itself. It alone makes sense, so (D) is correct.

 


 

8/21/24

 

Before Star Trek premiered on September 8, 1966, the show’s ingredients had been slow-cooking in creator Gene Roddenberry’s brain for years. At first, Roddenberry’s initial idea was to write a show about a 19th-century blimp that journeyed from place to place, making contact with distant peoples. Deciding instead to set the show in the future, Roddenberry drew upon his youthful immersion in science fiction magazines like Astounding Stories.

 

A. NO CHANGE
B. Roddenberry had the initial idea
C. Roddenberry’s idea initially was
D. Roddenberry’s idea was

 

To answer this question, you must back up to the beginning of the sentence. It contains the phrase At first, which is a synonym for initially. As a result, it is redundant to include both. Because the beginning of the sentence cannot be changed, initially must be removed from the underlined portion. That makes (D) correct.

 


 

8/20/24

 

Even before the advent of digital technologies, critics predicted the collapse of existing media. After television was invented, many claimed radio would die. But radio ended up surviving by finding new uses; people started listening in cars, during train rides and on factory floors.

 

Which of the following would NOT be an acceptable alternative to the underlined word?

 

A. destruction
B. demise
C. revolt
D. disappearance

 

Destruction, demise, and disappearance can all be used to indicate that critics believed existing media was on its way out. Revolt (rebellion) implies the opposite, however, so it is NOT an acceptable alternative to the underlined word. That makes (C) correct.

 


 

8/19/24

 

On March 19, 1918, Woodrow Wilson signed the Calder Act, requiring people in the United States to set their clocks to standard time; less than two weeks later, on March 31, they would be required to abandon standard time and pushed their clocks ahead by an hour for the nation’s first experiment with daylight saving.

 

A. NO CHANGE
B. push
C. have pushed
D. had pushed

 

The presence of the word and before the underlined verb signals a parallel construction: they would be required to (1) abandon standard time and (2) x. There is no option that supplies the infinitive (to push); however, the to before abandon can “apply” to the underlined verb as well. As a result, the verb alone can be used. That makes the answer (B).

 


 

8/18/24

 

In an exhibition called Figuring History, the African-American artist Robert Colescott provided a tongue-in-cheek send-up of the famous depiction of George Washington crossing the Delaware. The Oakland, California, native places George Washington Carver, the agricultural pioneer at the Tuskegee Institute in Alabama, in the spot of his namesake.

 

A. NO CHANGE
B. artist, Robert Colescott
C. artist Robert Colescott,
D. artist, Robert Colescott,

 

When commas are tested with names that appear in the middle of a sentence, there are typically only two correct options: no commas (essential) or two commas (non-essential). The easiest way to determine which is correct is to treat the name like a non-essential item and cross it out of the sentence. If the sentence still makes sense in context, the information is not essential, and commas are required; if the sentence does not make sense in context, the information is essential, and no commas should be used. Crossed out: In an exhibition called Figuring History, the African-American artist…provided a tongue-in-cheek send-up of the famous depiction of George Washington crossing the Delaware. Athough the sentence that remains makes grammatical sense, it omits a very important piece of information: we no longer know who the African-American artist is. As a result, the reference to The Oakland, California, native in the following sentence does not make sense. The information is therefore essential, and no commas should be used, making the answer (A).

 


 

8/17/24

 

The Brooklyn Academy of Music (BAM) has been the site of many creative adaptations of Shakespeare’s plays. Some are a multimedia mashup of characters, lines and scenes from Shakespeare’s history plays. “Extensively cut,” “deeply cut” and “severely cut” are some of the favorite phrases used by the reviewers of these types of experimental stage and film adaptations. The job can involve rearranging scenes, simplifying plotlines, and eliminating characters. In such cases, cutting up Shakespeare is not an act of destruction but an act of creation. Professional playwrights in Shakespeare’s time even thought about creating scripts as “cutwork,” like constructing costumes by cutting and stitching.

 

Which answer creates the most logical transition between the preceding sentence and the information that follows?

 

A. NO CHANGE
B. During Shakespeare’s time, it was not uncommon for multiple versions of plays to circulate.
C. Multimedia websites also offer contemporary “translations” of Shakespeare’s plays, along with notes and interviews.
D. Cutting, however, doesn’t necessarily mean getting rid of something.

 

Although the question is phrased in terms of a “transition,” you must focus on the information that follows because the underlined sentence must set up that information — what comes before is less important. The sentence after states that cutting up Shakespeare is not an act of destruction but an act of creation. Logically, then, the underlined sentence must be consistent with the idea that cutting Shakespeare is NOT about destroying his work. Be careful with (A): if you focus on the previous sentence, this answer might seem to fit with the general discussion of cutting, but the references to simplifying plotlines and eliminating characters are not really consistent with the idea of creation. (B) and (C) are simply off-topic. (D) is correct because the statement that cutting Shakespeare’s work is NOT about getting rid of something leads naturally into the idea that cutting = creation.

 


 

8/16/24

 

To some extent, the fear of having a book or movie plot “spoiled” is well-grounded. You only have one opportunity to learn something for the first time. Once you’ve learned it, that knowledge affects what you notice, what you anticipate, and even what your imagination can do.

 

A. NO CHANGE
B. what you can imagine.
C. what is in your imagination.
D. what your imagination is like.

 

The underlined portion involves the third item in a list, so the format of this item must match the format of the previous two. In most cases, you can determine the answer by focusing on the beginning of each item, but in this case all of the answers begin the same way, so you must consider the information that follows. In each of the first two items, what is followed by you + verb, so the third item must contain you + verb as well. (D) contains your rather than you, so it can be eliminated. In (C), a verb does follow what, but it also contains a prepositional phrase afterward, unlike the first two items. (B) contains two verbs (can and imagine), making it the closest match and thus the answer.

 


 

8/15/24

 

Long before smartphones filmed the stiffened appendages of people seeking internet fame, striking a pose was a popular form of entertainment in Victorian England. They called the practice “tableaux vivants” (literally, “living pictures”). The technique had its roots in medieval drama, but it became a fashionable Victorian-era dinner party game similar to charades. People would select a famous scene and position themselves in it, frozen, for their guests and friends to observe.

 

A. NO CHANGE
B. Participants
C. One
D. We

 

Although the general meaning of the sentence is clear in the original version, the pronoun they is vague and ambiguous — the sentence does not indicate who called the practice “tableaux vivants.” One and we create the same problem. Only (B) provides a noun, eliminating the ambiguity.

 


 

8/14/24

 

Paradoxically, time is perceived to pass slowly in situations in which there is either nothing happening or a great deal is happening. In other words, the complexity of the situation is either much higher or much lower than normal.

 

A. NO CHANGE
B. a great deal happens.
C. a great deal has happened.
D. a great deal happening.

 

The sentence in which the underlined portion appears describes two alternative scenarios, connected grammatically by the word or. Because the construction of the first item cannot be changed, the second must be made to match the first. The second item must also be able to follow there is. The first item contains nothing + -ING, so to remain parallel, the second item must contain a great deal + -ING. It is also correct to say time is perceived to pass slowly in situations in which there is…a great deal happening. Only (D) fits.

 


 

8/13/24

 

Since April 2017, a canoe powered solely by solar energy travels back and forth along the 42-mile stretch of the Capahuari and Pastaza rivers that connect the nine isolated settlements that live along their banks. The boat, named Tapiatpia after a mythical electric eel in the area, is the Amazon’s first solar powered public transport system.

 

A. NO CHANGE
B. has traveled
C. would travel
D. traveled

 

The word since is a tip-off that the present perfect (has/have + past participle) is required: this tense indicates that an action began in the past and is continuing into the present. (B) contains the correct construction, making it the answer.

 


 

8/12/24

 

Protection from ultraviolet (UV) rays is nothing new: many organisms, including microbes, plants, and animals, have developed the ability to shield themselves by producing small molecules that absorb UV rays, and block radiation, from entering cells and damaging the DNA.

 

A. NO CHANGE
B. molecules, that absorb UV rays and block radiation
C. molecules that absorb UV rays and block radiation
D. molecules that absorb UV rays and block radiation,

 

(A) and (D) can both be eliminated right away because they place a comma before a preposition (from), and (B) can be eliminated because as a rule, no comma should be placed before that. (C) is correct because no punctuation is required in the underlined portion.

 


 

8/11/24

 

Deep twilight settles in over Wales, Alaska. As the last traces of sunset orange give way to blue black on the western horizon, the icy Bering Strait and Siberia beyond are invisible in the night. All is quiet in the tiny village—a cluster of buildings with a single string of streetlights, tucked between frozen hills and frozen sea.

 

Which of the following is the LEAST acceptable alternative to the underlined portion?

 

A. village, a cluster
B. village: a cluster
C. village and a cluster
D. village; it is a cluster

 

The question asks you to identify the LEAST acceptable option, so the correct answer must be wrong. (A) is acceptable because a comma is used to set off a dependent clause that modifies the independent clause before it. (B) is acceptable because the colon is preceded by a complete sentence, and the information that follows is an explanation of what the tiny village is made up of. (D) is acceptable because the addition of it is at the beginning of the second clause makes that clause independent, and a semicolon is correctly used to separate it from the independent clause that comes before it. (C) is NOT acceptable because the word and prevents the information after the comma from describing the tiny village. This answer implies that a cluster of buildings, etc. is a separate location from the tiny village, a meaning not implied by the original construction in the passage. As a result, (C) is the LEAST acceptable answer and is thus correct.

 


 

8/10/24

 

Unlike his peers, the architect Frank Lloyd Wright also had a rare artistic passion that was very unusual: Japanese art. Wright first became interested in his early twenties, and within a decade, he was an internationally known collector of Japanese woodblock prints.

 

A. NO CHANGE
B. a rarely unusual artistic passion
C. a rare and unusual artistic passion
D. a rare artistic passion

 

Something that is rare is by definition unusual, so it is redundant to use both words. (D) is the only option that includes only one of these terms, so it is correct. Shortcut: shorter is better. On a purely visual basis, you can start by assuming that (D) is right.

 


 

8/9/24

 

Spam has become a sought-after product in many countries around the world since its introduction in the 1930s, especially those that have faced economic hardship. Because it’s cheap, filling, and long-lasting, it addresses a genuine need.

 

A. NO CHANGE
B. Since its introduction in the 1930s, Spam has become a sought-after product in many countries around the world,
C. In many countries around the world, Spam has become a sought-after product since its introduction in the 1930s,
D. Around the world, in many countries, Spam has become a sought-after product since its introduction in the 1930s,

 

The key to answering this question is to focus on the information that comes after the underlined portion. The phrase especially those that have faced economic hardship can only refer to many countries around the world — if another phrase, e.g., in the 1930s, is placed at the end of the underlined portion, a misplaced modifier is created. As a result, the correct answer must end with the phrase many countries around the world. (B) is the only answer to contain that construction, so it is correct.

 


 

8/8/24

 

In the 1840s, the travel writer Alexander Mackay described the “extraordinary number” of newspapers that travelers would encounter everywhere they went. Henry David Thoreau, on the other hand, was more appalled than dazzled. In fact, he loathed newspapers, denouncing them for a variety of offenses, including “servility” and outright baseness.

 

A. NO CHANGE
B. However,
C. Likewise,
D. Subsequently,

 

Start by ignoring the transition already in the passage, and focus on determining the relationship between the sentence begun by the underlined transition and the sentences before. The preceding portion of the passage indicates that Thoreau was more appalled (horrified) than dazzled by the plethora of newspapers, and the sentence begun by the transition indicates that he loathed and denounced them. Those are similar ideas, so a continuer is required. That eliminates (B). (C) doesn’t quite fit: likewise is used to indicate that two distinct examples support the same point, or to introduce a new, similar idea—it is not used to continue the same point, as is the case here. (D) is incorrect because subsequently is a synonym for next, or afterward, and the transition is not introducing a new step in a sequence. In fact is correct because the statement begun by the transition serves to emphasize and expand on the idea that Thoreau was more appalled than dazzled. That makes the answer (A).

 


 

8/7/24

 

The more often people hear a statement, the more likely they are to believe it’s true—a phenomenon commonly known as the illusory truth effect. Adding a picture can also change how believable a statement is. Sometimes, images can make messages more convincing; other times, skepticism is increased.

 

A. NO CHANGE
B. skepticism would be increased.
C. there is an increase in skepticism.
D. they can increase skepticism.

 

The construction [s]ometimes…other times in the last sentence indicates that the second half of the sentence must be parallel in structure to the first. The first half begins with subject + can + verb, so the second half must match. (D) is the only answer to contain that construction (they can increase), so it is correct.

 


 

8/6/24

 

When he published The Sun Also Rises in 1926, Ernest Hemingway was already well-known among expatriate writers in Paris and cosmopolitan literary circles in New York and Chicago. However, it was his second novel A Farewell to Arms, that truly made him a celebrity. With this newfound fame, Hemingway learned, came fan mail, and lots of it.

 

A. NO CHANGE
B. novel: A Farewell to Arms
C. novel, A Farewell to Arms,
D. novel, A Farewell to Arms

 

The easiest answers to eliminate are (B) and (D). (B) is incorrect because a colon must follow a complete, standalone sentence, and However, it was his second novel clearly is not a sentence. (D) is incorrect because when a title appears in the middle of a sentence, it is always wrong to place a single comma before it. (A) and (C) both appear to violate a comma rule; however, when these answers are plugged into the sentence, the comma after Arms results in the seemingly incorrect placement of a comma after the word that. The only way that this construction can be made acceptable is if the title is made non-essential (two commas). When it is crossed out, the underlying structure of the sentence makes sense: However, it was his second novel…that truly made him a celebrity. In contrast, the placement of only a single comma before that creates an unnatural break. (A) can thus be eliminated, making (C) correct.

 


 

8/5/24

 

In the late 1970s, a group of researchers set out testing the improbable idea of making computers “talk” to one another by using digital information packets that could be traded among multiple machines. The project, called ARPANET, went on to fundamentally change life on Earth under its more common name: the Internet.

 

A. NO CHANGE
B. in testing
C. for testing
D. to test

 

The correct idiom is set out + infinitive (to test); the gerund (-ING) form is incorrect, regardless of whether a preposition is used before it. That makes (D) the only possible answer.

 


 

8/4/24

 

In the nineteenth century, people in the United States ate dessert puddings that still are recognizable today; however, they also ate main-course puddings like steak and kidney pudding, pigeon pudding, or eating mutton pudding, in which stewed meats were surrounded by a flour or potato crust. Other puddings had no crust at all. Some, like Yorkshire pudding, were a kind of cooked batter.

 

A. NO CHANGE
B. they ate
C. ate
D. DELETE the underlined word.

 

The underlined portion involves the third item in a list, so the format of this item must match the format of the previous two. The first two items contain nouns only (steak and kidney pudding, pigeon pudding), so the third item must contain only a noun as well. (A), (B), and (C) are incorrect because they contain other parts of speech. (D) creates the correct construction by deleting the verb, leaving only the noun mutton pudding.

 


 

8/3/24

 

You are invited into Do Ho Suh’s apartment. You put down your bag, remove your coat and step inside. The hallway changes color as you proceed, first pink, then green and then blue. There is a red staircase outside, and beyond it people are moving around. You can see them right through the walls. Back home, the only things that behave this way are cobwebs, but here, everything—door panels, chain locks, light switches, sprinkler system dissolves delightfully into colored light.

 

A. NO CHANGE
B. chain locks; light switches, sprinkler system,
C. chain locks, light switches sprinkler system
D. chain locks, light switches, sprinkler system—

 

The list door panels, chain locks, light switches, sprinkler system is non-essential because the sentence still makes sense when it is removed: Back at your house, the only things that behave this way are cobwebs, but here, everything…dissolves delightfully into colored light. As a result, a second dash must be used to mark the end of the non-essential clause. That makes (D) the only possible answer. In the other answers, the various types of punctuation within the list are only a distraction.

 


 

8/2/24

 

Researchers have reported that individuals, who live in urban areas of more than half a million inhabitants, are exposed to night-time light levels three to six times brighter than those in small towns and rural areas. People living in regions with more intense light sleep less, are more tired during the daytime, and report feeling more dissatisfied with their sleep.

 

A. NO CHANGE
B. individuals, who live in urban areas of more than half a million inhabitants
C. individuals who live in urban areas of more than half a million inhabitants,
D. individuals who live in urban areas of more than half a million inhabitants

 

When commas with “who” clauses in the middle of a sentence are tested, there are typically only two possible answers: two commas (non-essential) or no commas (essential). Answers with a comma only before the “who” clause are always wrong, and answers with a comma only after the “who” clause are, with a few rare exceptions, wrong as well. In this case, the clause is essential because the sentence is not talking about individuals in general, as two commas would imply, but rather about a specific group of individuals: those who live in urban areas with more than half a million inhabitants. In other words, the meaning of that clause is restricted to that particular group. No commas indicate a restricted meaning, so (D) is correct.

 


 

8/1/24

 

The Museum of Bad Art was founded in 1994, when Boston art and antique dealer Scott Wilson rescued a portrait of a handsome grandmother, pensively poised under an aggressively yellow sky in a windswept meadow, from a Boston trash heap. Wilson wanted to sell the frame, but upon seeing the painting (later dubbed Lucy in the Field with Flowers), an objection was made by his friend Jerry Reilly. Reilly took the tribute to someone else’s elder and hung it in his own home.

 

A. NO CHANGE
B. Jerry Reilly, his friend made an objection.
C. his friend Jerry Reilly, who made an objection.
D. his friend Jerry Reilly objected.

 

(A) is incorrect because it contains a dangling modifier, albeit a hidden one. To simplify the sentence and reveal the error, cross out the information in parentheses: Wilson wanted to sell the frame, but upon seeing the painting, an objection was made by his friend Jerry Reilly. Who saw the painting? Jerry Reilly, not an objection. Because Jerry Reilly is not placed immediately after painting, a dangling modifier is created. Although (B) and (C) place Reilly’s name in the appropriate spot, these answers are both incorrect because they contain fragments — in neither case does the verb correspond to the subject, Jerry Reilly. (D) corrects this error and is shorter, clearer, and less awkward, so it is the answer.

 


 

7/31/24

 

For years, Stefan Strumbel, a street artist born and raised in the small city of Offenburg, Germany, has wrestled with the idea of “heimat”—a German word that translates loosely as “homeland”—and how his art should reflect it. That’s one reason, he says that he decided to stop painting graffiti and focus on cuckoo clocks instead.

 

A. NO CHANGE
B. reason he says, that
C. reason, he says, that
D. reason he says that,

 

As a general rule, a comma should not be placed before the word that; however, this question involves an exception. The most logical grammatical “interpretation” of the sentence is that he says is used as a non-essential clause because when that phrase is crossed out, the sentence still makes sense: That’s one reason…that he decided to stop painting graffiti…. As a result, a comma is required both before and after he says, making the answer (C). Otherwise, all of the other options create illogical breaks in the sentence.

 


 

7/30/24

 

Before publishing Silent Spring, the book that flung the modern environmental movement, Rachel Carson was a well-known author of beautifully descriptive books about marine life, including The Sea Around Us, which became a surprise hit after it was published in 1951.

 

A. NO CHANGE
B. hurled
C. launched
D. tossed

 

Saying that a movement was launched is another way of saying that it began. The other options are idiomatically unacceptable in this context. (C) is thus correct.

 


 

7/29/24

 

Because the Hollywood sign is so famous today, it may be surprising to learn that it wasn’t until fairly recently that it achieved its iconic status. In the 1930s and 1940s, however, the sign made an appearance in only a few of the movies that were about Hollywood or the movie industry. Other Hollywood institutions, like the Brown Derby restaurant, tended to represent the film world.

 

A. NO CHANGE
B. for example,
C. therefore,
D. likewise,

 

The underlined transition is placed between two commas in the middle of a sentence, so it serves to indicate the relationship between the sentence in which it appears and a previous statement — NOT two parts of the same sentence. With that information in mind, start by ignoring the original transition so that you don’t get distracted by it, and focus on the relationship between the information that comes before. The beginning of the passage states that the Hollywood sign didn’t become famous until fairly recently. The sentence that follows indicates that the Hollywood sign made an appearance in only a few films about the movie industry. Those are similar ideas, so (A) can be eliminated. (C) does not fit because the sign’s appearance in only a few films in the 1930s and ’40s is not a result of the fact that it didn’t become “iconic” until recently. (D) can also be eliminated because the sentence with the transition is not presenting a second example or situation that is similar to a first. Rather, the Hollywood sign’s infrequent appearance in old movies is cited as a single example of its relative obscurity. That makes (B) correct.

 


 

7/28/24

 

It turns out that water worlds may be some of the worst places to look for living things. One recent study shows how a planet covered in oceans could be starved of phosphorus, a nutrient without which earthly life cannot thrive. Other work concludes that a planet swamped in even deeper water would be geologically dead, lacking any of the planetary processes that nurture life on Earth.

 

A. NO CHANGE
B. it
C. this
D. DELETE the underlined word.

 

(A) is correct because which is the only option that creates a sentence that is both grammatical and logical. Replacing which with it creates a jumbled, nonsense clause (a nutrient without it earthly life cannot thrive), eliminating (B). (C) does not work grammatically or logically. The sentence consists of an independent clause (One recent study shows how a planet covered in oceans could be starved of phosphorus) followed by a comma; in order to avoid a comma splice, the following clause must be dependent. Replacing which with this creates an independent clause (a nutrient without this earthly life cannot thrive) and thus a comma splice. In addition, this construction implies that phosphorous is this earthly life, a meaning that is somewhat bizarre. (D) is incorrect because the phrase after the comma no longer refers back to phosphorous if the underlined pronoun is removed (a nutrient without earthly life cannot thrive) — a pronoun is required to “tie” the two parts of the sentence together. (A) correctly provides that pronoun, making it the answer.

 


 

7/27/24

 

In 1934, Babe Ruth and his American teammates embarked on an 18-game tour of Japan. Swatting 13 home runs, waving American and Japanese flags, clowning with kids, and he even donned a kimono, the Babe won the hearts and minds of the Japanese people.

 

A. NO CHANGE
B. even donning
C. even to don
D. even don

 

The underlined section is part of the fourth item in a list. The previous items begin with -ING words (swatting, waving, clowning), so the final item must begin that way as well. Only (B) contains this construction, so it is correct.

 


 

7/26/24

 

Our relationship with horses is distinct from our relationships with cats and dogs; horses sit at the intersection of being wild and domesticated and don’t fit easily into the category of pet. Perhaps this difference also has to do with its large size, which creates an element of danger.

 

A. NO CHANGE
B. it’s
C. their
D. they’re

 

To identify the noun to which the underlined pronoun refers (i.e., the antecedent), you must back up to the previous sentence. The only noun that fits is horses, plural — it would not make sense for its to refer to the singular noun difference. As a result, a plural pronoun is required, eliminating (A) and (B). (D) is incorrect as well because you would not say, Perhaps this difference also has to do with they are large size. That leaves (C), which correctly provides the plural possessive, their. Their large size = the large size of horses.

 


 

7/25/24

 

Recently, I did something that many people would consider unthinkable, or at least very strange. Before going to see a movie, I deliberately read a review that revealed all of the major plot points, from start to finish.

 

A. NO CHANGE
B. movie, I deliberately: read a review that
C. movie I deliberately read a review, that
D. movie, I deliberately read a review

 

(B) is incorrect because a complete, standalone sentence must be placed before a colon, and Before going to see a movie, I deliberately is clearly not a complete sentence. (C) is incorrect because as a general rule, no comma should be placed before the word that. (D) makes sense on its own, but the removal of that creates a nonsense construction when this answer is plugged into the passage: Before going to see a movie, I deliberately read a review revealed all the major plot points... (A) is correct because it places a comma between a dependent clause (Before going to see a movie) and an independent clause (I deliberately read a review that revealed all the major plot points, from start to finish), without adding any unnecessary punctuation.

 


 

7/24/24

 

For centuries people set their clocks and watches by looking up at the sun and estimating, a tradition that led to wildly dissimilar results between (and often within) cities and towns. To railroad companies around the world, that wasn’t acceptable. They needed synchronized, predictable station times for arrivals and departures, so they proposed splitting up the globe into 24 time zones.

 

Which of the following would NOT be an acceptable alternative to the underlined word?

 

A. produced
B. created
C. resulted
D. yielded

 

Be careful when plugging in the options: produced, created, and yielded all fit, as does resulted in; however, (C) only includes the verb, and it is idiomatically incorrect to say a tradition that resulted wildly dissimilar results. Because the question asks you to identify the answer that is NOT acceptable, (C) is correct.

 


 

7/23/24

 

When the radio became prevalent in the 1930s, Orson Welles perpetrated a famous hoax about extraterrestrials with his infamous “War of the Worlds” program. This broadcast didn’t actually cause widespread fear of an alien invasion among listeners, as some have claimed; however, they did spark a national conversation about mass media and audience gullibility.

 

A. NO CHANGE
B. it
C. these
D. DELETE the underlined word.

 

The answer choices contain both singular and plural pronouns, indicating that the question is testing pronoun agreement. As a result, you must start by identifying the underlined pronoun’s antecedent — that is, the noun to which it refers. Although the first half of the sentence does contain a plural noun (listeners), this does not make sense as the antecedent of the underlined word — listeners did not spark a national conversation. Both they and these are plural, so (A) and (C) can be eliminated. (D) does not work because the underlined pronoun cannot be deleted without creating a nonsense construction: a semicolon must be followed by a complete sentence, and removing the pronoun means removing the subject. (B) is correct because the logical antecedent of the underlined pronoun is the singular noun broadcast. (What sparked a national conversation about media and mass gullibility? This broadcast.)

 


 

7/22/24

 

Based in Mexico City, the artist known only as Curiot is famous for his colorful paintings featuring mythical half-animal, half-human figures. Most of which are rooted in Mexican tradition and depicted with meticulous detail, geometrical patterns, and vibrant hues.

 

A. NO CHANGE
B. figures. Most of these creatures
C. figures, with most of these creatures
D. figures, most of them

 

(A) is incorrect because pronoun + of which (most of which) signals a dependent clause and cannot be used to begin a sentence. (C) is incorrect because the idiomatic structure is with…-ING (e.g., with most of these figures being rooted); a conjugated verb (are) cannot be used instead. (D) is incorrect because pronoun + of them signals an independent clause and thus cannot be placed after a comma without creating a comma splice. (B) corrects all of these errors by placing a period between two complete sentences.

 


 

7/21/24

 

I don’t know why that particular photo of a half-finished sweater caught my attention, but as soon as I saw it, I wanted to learn to knit. At first, I wasn’t sure I needed another hobby, but after I read an essay by Ann Hood, “Ten Things I Learned From Knitting,” the decision was made by me.

 

A. NO CHANGE
B. my decision was made by me.
C. my decision had been made.
D. I made my decision.

 

(A), (B), and (C) are all incorrect because they use the passive voice (x is done by y) to create unnecessarily wordy and awkward constructions. (C) is also wrong because the past perfect (had been + verb) is used to indicate a completed action that came before a second action, but the passage clearly indicates that the narrator made the decision after reading Ann Hood’s essay. (D) is correct because it uses the active voice and is cleaner and more concise than the other options. In addition, an active construction is used earlier in the sentence (after I read an essay…), and this answer maintains that syntax.

 


 

7/20/24

 

Located in Abu Dhabi, the new branch of the Louvre Museum is intended to look like a floating dome: the glimmering structure appears to hover over the sparkling water that surrounds it, and its webbed pattern allows the sky to filter through. The overall effect is meant to evoke rays of sunlight passing through palm leaves in a desert oasis.

 

If the writer were to delete the words glimmering and sparkling from the underlined portion, the sentence would most nearly lose details that:

 

A. emphasize the effects of light on the museum building.
B. highlight the contrast in appearance between the museum building and its surroundings.
C. provide an overview of the main sections of the museum.
D. make clear that water is the primary influence on the museum’s design.

 

To answer this question, you must take the end of the passage into account. The last sentence states that the museum’s design is intended to evoke rays of sunlight passing through palm leaves in a desert oasis—in that context, the use of words like glimmering and sparkling is intended to emphasize the importance of light, a fact that directly corresponds to (A). (B) is incorrect because the passage says/implies nothing about a contrast between the museum building and its surroundings, and (C) is incorrect because the passage also says nothing about the sections of the building. If you read only the underlined portion, (D) might seem plausible, but the last sentence directly contradicts the idea that water is the primary influence on the design of the building.

 


 

7/19/24

 

Unlike the Venus flytrap, the lobes or leaves of the waterwheel do not change shape when they snap shut; rather, closing like two halves of a mussel shell. In contrast, the Venus flytrap flexes its leaves from flat to curved when enclosing its prey.

 

A. NO CHANGE
B. shut, but rather closing
C. shut but rather close
D. shut but rather closes

 

(A) is incorrect because a complete sentence must follow a semicolon, but rather, closing like two halves of a mussel shell is not a sentence. (B) is incorrect because a conjugated verb rather than an -ING word (gerund) must be used in order to maintain parallel structure with change. (D) is incorrect because the subject of the underlined verb is lobes or leaves (plural), whereas closes is singular. (C) is correct because it provides a conjugated plural verb (close).

 


 

7/18/24

 

There are almost 90 million cats in the United States, or one for every three households. That makes cats more popular, petwise, than dogs. The majority of them—about two-thirds to three-fourths, surveys say—are sweet, harmless, cuddly housecats, which seldom set foot outside. The other one-quarter to one-third aren’t so harmless. Equipped with laser-quick paws and razor-tipped claws, they are the stuff of every bird and small mammal’s nightmares.

 

A. NO CHANGE
B. three-fourths, surveys say, are sweet
C. three-fourths, surveys say, are sweet—
D. three-fourths—surveys say are sweet,

 

Between all of the dashes and internal hyphens, this question is somewhat confusing visually, but the presence of a dash before the underlined section, coupled with the fact that three of the answers contain dashes, is a big clue that a non-essential clause is involved here. As a result, it is easiest to start by identifying that clause. One helpful shortcut is to know that non-essential clauses typically end before verbs, and here there is only one option that places the second dash before a verb: (A). To confirm that answer, try crossing out the information between the dashes: The majority of them…are sweet, harmless, cuddly housecats, which seldom set foot outside. Yes, that makes sense, so (A) is correct.

 


 

7/17/24

 

Announced in 2013, the BRAIN Initiative is a massive project undertaken by a group of agencies and individuals, including universities, technology companies, and neuroscientists. The Initiative includes a variety of programs designed to lower the barriers between the human brain and the digital world, with the goal being to understand how the brain processes information.

 

A. NO CHANGE
B. to be
C. has been
D. is

 

The idiomatic construction is with…-ING, which can be used as an alternative to and (with the goal being to understand = and the goal is to understand). As a result, being, which normally signals a wrong answer, must be used. That makes (A) correct.

 


 

7/16/24

 

Training to go to Mars requires a substantial suspension of disbelief. But that has not stopped scores of people from participating in simulations that re-create Mars on Earth in order to better understand and prepare for the challenges of one day sending humans to the red planet. Often set in dusty, remote locations, these so-called Mars analogs often feature lifestyle choices meant to approximate humanity’s journey to the next planet over. There’s a 20-minute communications delay (no phone calls); freeze-dried meals; and limited water supplies. Moreover, participants can never leave the habitat without a spacesuit on.

 

The writer is considering deleting the underlined portion of the sentence. Should the writer do this?

 

A. Yes, because it provides background information that is irrelevant to the main focus of the paragraph.
B. Yes, because it suggests that people will travel to Mars in the near future.
C. No, because it provides an explanation for why people choose to participate in simulations of life on Mars.
D. No, because it describes some of the challenges involved in traveling to Mars.

 

When a question asks whether a given portion of a sentence should be deleted, it’s really asking whether the information in that section is on- or off-topic. What is the passage about? Simulations of life on Mars. What is the focus of the underlined portion? Why people participate in these “Mars analogs,” or the purpose behind Mars analogs. Is that information relevant to the passage? Yes, so (A) and (B) can be eliminated. (D) does not fit because the underlined portion has nothing to do with the challenges involved in traveling to Mars. That leaves (C), which correctly indicates that the purpose of the underlined portion is to explain why people spend months pretending to live on Mars.

 


 

7/15/24

 

Because laws prohibiting excess noise failed to satisfy people’s desire for quiet products and technologies emerged to meet the demand of increasingly sensitive consumers. In the early twentieth century, sound-muffling curtains, softer floor materials, room dividers, and ventilators kept the noise from the outside from coming in, while preventing sounds from bothering neighbors.

 

A. NO CHANGE
B. quiet, products and technologies
C. quiet products, and technologies
D. quiet products and technologies,

 

(A) is incorrect because the lack of a comma after quiet results in this word modifying products and technologies and creates one very long clause that begins with a conjunction (because) and thus cannot stand on its own as a sentence. (C) is incorrect because no comma should be placed between compound nouns (two nouns joined by and). To test this answer out, you can also think of the rule this way: comma + and = period, but a period cannot be plugged in after products because the preceding clause is not a complete sentence (Because laws prohibiting excess noise failed to satisfy people’s demand for quiet products.) (D) is incorrect because this answer places a comma between a subject (products and technologies) and the verb that follows (emerged). (B) is correct because the comma after quiet serves to break the sentence into two clauses: a dependent clause (Because laws prohibiting excess noise failed to satisfy people’s demand for quiet) and an independent clause (products and technologies emerged to meet the demand of increasingly sensitive consumers) that together form a complete sentence.

 


 

7/14/24

 

There are over 50,000 therapy dogs in the United States, and they’re becoming more popular in countries from Norway to Brazil. Trained and certified by a variety of organizations, hospitals and other facilities welcome these dogs and their handlers, who interact with patients.

 

A. NO CHANGE
B. Trained and certified by a variety of organizations, these dogs and their handlers interact with patients and are welcomed by hospitals and other facilities.
C. These dogs and their handlers, welcomed by hospitals and other facilities and trained and certified by a variety of organizations.
D. Welcomed by hospitals and other facilities, these dogs and their handlers being trained and certified by a variety of organizations.

 

(A) is incorrect because it contains a dangling modifier: the phrase Trained and certified by a variety of organizations must logically describe these dogs and their handlers, but the latter phrase does not appear immediately after the former. (C) is incorrect because it is a fragment: this answer does not contain a main clause that can stand on its own as a sentence. (D) is also a fragment because it contains a gerund (being) rather than a main verb. (B) corrects the dangling modification by placing these dogs and their handlers after the introductory phrase and does not introduce any new errors.

 


7/13/24

 

Like many successful authors of the nineteenth century, Washington Irving struggled against literary bootleggers. In England, some of his sketches were reprinted in periodicals without his permission, a legal practice since there was no international copyright law at the time. To prevent further piracy in Britain, Irving paid to have the first four American installments published as a single volume by John Miller in London.

 

Which of the following would NOT be an acceptable alternative to the underlined portion?

 

A. permission—a legal practice
B. permission; that practice was legal
C. permission, and a legal practice
D. permission, a practice that was legal

 

To answer this question, focus on whether information after the punctuation in each answer choice (dash, semicolon, comma) is a complete sentence because that factor determines what type of punctuation can and cannot be used. (A) is acceptable because a legal practice since there was no international copyright law at the time is a fragment, and a dash can correctly come before a fragment. (B) is acceptable because that practice was legal since there was no international copyright law at the time is a complete sentence, and a complete sentence must follow a semicolon. (D) is acceptable because a practice that was legal since there was no international copyright law at the time is a fragment, which can correctly follow a comma. Although (C) contains a comma as well, that comma is followed by and — and comma + and = period. If you plug in a period, you get A legal practice since there was no international copyright law at the time, a statement that is clearly not a sentence. Because the question asks you to identify the option that is NOT acceptable, (C) is correct.

 


 

7/12/24

 

The older you get, the more difficult it is to learn to speak French like a Parisian. However, no one knows exactly what the cutoff point is—at what age it becomes harder, nevertheless, to pick up noun-verb agreements in a new language.

 

A. NO CHANGE
B. for instance,
C. moreover,
D. likewise,

 

To simplify the sentence and avoid getting distracted by the transition already in the passage, ignore the transition and re-read the passage without it. The passage is discussing the age at which it becomes more difficult for people to learn a new language. In that context, [picking] up noun-verb agreements is an example of a skill that becomes more difficult as people get older. The only option to convey that relationship is for instance, which indicates that an example is being presented. (B) is thus correct.

 


 

7/11/24

 

They call it “the Never-Ending Storm of Catatumbo” or “The Lighthouse of Maracaibo”: something so familiar that people in the state of Zulia in Venezuela even put it on their flag. Less than half an hour after the first cloud forms, it starts to flash. It does this faster and faster — 200 times a minute is not uncommon. Afterward, the cloud becomes a giant bulb that lights up the night.

 

Which choice provides new information that is relevant to the rest of the paragraph?

 

A. NO CHANGE
B. lightning
C. a storm
D. a natural force

 

The key phrase in the question is “relevant to the rest of the paragraph.” As a result, it is necessary to consider the information in the rest of the passage when determining the answer. What is the focus of the rest of the passage? Something that starts to flash, and that is a giant bulb that lights up the night. The only option that corresponds to that description is “lightning,” so the answer is (B).

 


 

7/10/24

 

Crossword puzzles are said to be the most popular and widespread word game in the world, yet they have a relatively short history. The first crosswords appeared in British children’s books during the nineteenth century, they were simple games, apparently derived from the word square: a group of words arranged so that the letters read alike vertically and horizontally. In the United States, however, the puzzle developed into a serious adult pastime.

 

A. NO CHANGE
B. Although the first crosswords
C. Until the first crosswords
D. When the first crosswords

 

To answer this question, you must consider the sentence as a whole; the underlined section alone does not give you enough information to determine the correct option. When considered independently, the underlined portion makes sense; however, the construction comma + they later in the sentence (nineteenth century, they were simple games) signals a comma splice — two complete sentences separated by a comma. Because comma + they cannot be changed, a conjunction must be added to the beginning of the sentence to make the first clause dependent. Although does not make sense: the first crosswords were not simple games despite the fact that they appeared in children’s books. Those are similar ideas, not different ones. Until also does not make sense: this answer would imply that crosswords were simple games before they appeared in children’s books (at which point they became more complex). Only when fits: it indicates that crossword puzzles were simple at the time they appeared in children’s books. That makes (D) correct.

 


 

7/9/24

 

When German immigrants first started coming to the United States in the 1700s, they brought the pretzel with them. Bavarians and other southern Germans had been enjoying pretzels for hundreds of years. Sometimes they ate pretzels as a side to a main dinner course; other times, they chowed down on sweet pretzels for dessert. In Swabia, a region in southwestern Germany, signs for bakeries still include gilded pretzels hanging over the door.

 

A. NO CHANGE
B. chomped on
C. consumed
D. chugged

 

Chowed down on, chomped on, and chugged are all excessively slangy and casual when compared to the types of words used in the rest of the passage. In addition, chugged can only describe something done to a liquid, not a solid food. Only consumed is consistent with the moderately serious tone of the passage, making the answer (C).

 


 

7/8/24

 

While most paintings produced by members of the Hudson River School were rendered realistically, many of the scenes they depicted were synthesized from a variety of natural images observed by the artists. In gathering the visual data for their paintings, the artists would travel to environments with extraordinary and extreme conditions that did not permit extended painting in these environments. During the expeditions, the artists recorded sketches and memories, returning to their studios to paint the finished works later.

 

A. NO CHANGE
B. in such environments.
C. in such places.
D. DELETE the underlined portion.

 

To answer this question, you must take the entire sentence into account — the underlined portion does not provide enough information to answer the question. Before the underlined section, the writer already makes clear that artists would travel to environments with extraordinary and extreme conditions that did not permit extended painting. As a result, it is redundant to restate this information. (Although (C) replaces environments with places, the result is still the same.) The information should therefore be deleted, making (D) correct.

 


 

7/7/24

 

Key-Sook Geum is an artist, fashion designer, and scholar from the Republic of Korea. Having taught and worked in fashion design, Geum combines art with fashion through her exquisite wire sculptures in the shape of women’s clothing.
She is inspired by the shapes and styles of clothing from Korea’s Joseon Dynasty (1392–1910). The items tell stories about the people who wore them: their lives, values, and beliefs.

 

A. NO CHANGE
B. sculpture’s in the shape of womens
C. sculptures in the shape of womens’
D. sculptures’ in the shape of women’s

 

As a rule, a noun can only be possessive when it is followed by another noun, so start by looking at the words after sculptures and women’s. Sculptures is followed by in, which is a preposition rather than a noun, so this word cannnot be possessive (no apostrophe). That eliminates (B) and (D). The word clothing, which follows women’s, is a noun (you can say the clothing) so an apostrophe is necessary — the question is whether it belongs before or after the -s. Although the plural possessive (women’s clothing = clothing belonging to women) is normally formed by adding -s + apostrophe to a noun, women is irregular: that is, its plural is not formed by adding -s to its singular form (woman). To form the possessive of an irregular plural noun, add apostrophe + -s (women’s). (A) is thus correct.

 


 

7/6/24

 

Located in London’s Kew Gardens, the greenhouse known as the Temperate House is home to a geographically arranged collection of 10,000 plants from temperate climates around the world. These areas are sometimes described as “the Goldilocks zone” of the planet. Plants are safe from frost there.

 

Which of the following choices most effectively combines the underlined sentences?

 

A. These areas are sometimes described as the “Goldilocks zone” of the planet, being that plants are safe from frost there.
B. These areas are sometimes described as the planet’s “Goldilocks zone,” where plants are safe from frost.
C. These areas are sometimes described as the “Goldilocks zone” of the planet, and that is an area where plants are safe from frost.
D. Described sometimes as the “Goldilocks zone” of the planet, plants are safe from frost there.

 

Shortcut: shorter is better. When you are asked to combine sentences, the shortest answer will frequently be correct, and you should start by looking at it. That is the case here: (B), the shortest answer, is also the cleanest and least awkward, and it does not contain any grammatical errors. Otherwise, (A) is incorrect because the phrase being that is wordy and awkward (note that answers with being are almost always wrong). (C) is incorrect because the phrase and that is an area where is unnecessarily wordy and repetitive. (D) is incorrect because it contains a dangling modifier: the phrase Described sometimes as “the Goldilocks zone” of the planet refers to areas, so that word, not plants, must follow the comma. (B) is correct because it joins the sentences cleanly, using only one word (where).

 


 

7/5/24

 

It’s difficult to describe how excited I was when two veteran mountain climbers asked me to join them for a winter attempt on Gasherbrum II, one of the tallest peaks in the Himalayas. Like any adventurous activity, mountaineering has hazards. They must find someone who can tolerate extremely challenging conditions—frostbite burns, intense hunger, the loss of feeling in fingers and toes, overwhelming weariness—all while maintaining the will to push forward.

 

Which of the following true statements provides the most appropriate transition between the previous sentence and the information that follows?

 

A. NO CHANGE
B. Mountaineers must choose their climbing partners with extreme care.
C. Personal preparedness and skill development are very important.
D. Instructors teach many skills, including the fundamentals of survival in a cold environment.

 

Although this question is phrased in terms of transitions, the easiest way to answer it is to focus on the information that follows the underlined statement. That statement must logically set up the information that follows. In this case, the following sentence refers to the fact that [climbers] must find someone who can tolerate some very challenging conditions. Logically, the underlined sentence must indicate who that “someone” who must be found is — otherwise, the following sentence does not make sense. The choices that refer to specific people are (B) (mountaineers) and (D) (instructors). (B) is correct because mountaineers must logically choose partners who can withstand such difficult conditions. Given that the narrator states that s/he was invited to join two veteran mountain climbers, the reference to instructors does not make sense — the narrator is describing a partnership, not a student/teacher scenario.

 


 

7/4/24

 

Many marine animals are large, rare, elusive, and highly mobile. Sharks are an obvious example: in the oceans they make up a small proportion of the biomass, are difficult to catch, and they have been in conflict with humans for thousands of years.

 

A. NO CHANGE
B. have been
C. having been
D. being

 

The underlined portion involves the third item in a list, so the format of this item must match the format of the previous two. (C) and (D) do not match either of the first two items at all and can be eliminated right away. Be careful with (A): although the construction pronoun + verb (they have been) is the same as in the first item (they make up), this answer does not match the second item, which begins with a verb alone (are). As a result, this answer is incorrect. Although (B) may appear to contain the same problem, in fact it is acceptable. The pronoun they before the first item can in fact “apply” to the verbs in the following items, making it unnecessary to repeat the pronoun. (B) is thus correct.

 


 

7/3/24

 

We know a lot about carbon, the element that forms the chemical backbone of life, in our crust and oceans. We know far less about it in the Earth’s core and mantle. So far, it’s proved challenging to sample the mantle, which extends up to 1,800 miles below the surface and plays a huge yet mysterious role in the global carbon cycle.

 

A. NO CHANGE
B. surface, and plays a huge yet mysterious,
C. surface and plays a huge, yet mysterious
D. surface, and plays a huge yet mysterious,

 

Comma + and = period, so plug in a period in place of and: So far, it’s proved challenging to sample the mantle, which extends up to 1,800 miles below the surface. Plays a huge yet mysterious role in the global carbon cycle. Clearly, the second statement is not a sentence, so the comma before and is incorrect. That eliminates (B) and (D). (C) is also incorrect because when two adjectives are separated by but or yet, no comma should be used between them. (A) is correct because no punctuation is necessary in the underlined section.

 


 

7/2/24

 

If you’ve heard the term “grazer” before, it may bring to mind familiar farm animals, such as cows or sheep munching on pastureland. But the ocean has its own suite of grazers, one with very different — even bizarre — body forms and feeding techniques.

 

A. NO CHANGE
B. animals such as cows or sheep,
C. animals such as cows or sheep
D. animals such as: cows or sheep

 

(A) is incorrect because the comma before such would logically seem to begin a non-essential phrase (such as cows or sheep), but there is no second comma to end the phrase. (B) contains the opposite error: a comma is placed at the end of the non-essential phrase, but there is no comma at the beginning. (D) is incorrect because a colon must be placed after a complete, standalone thought, and If you’ve heard the term “grazer” before, it may bring to mind familiar farm animals, such as clearly cannot stand on its own as a sentence. Although the phrase such as cows and sheep can be treated non-essentially, it does not need to be. As a result, the commas are optional; it is also acceptable not to use any punctuation. (C) is thus correct.

 


 

7/1/24

 

Five decades into his life, Phineas Taylor Barnum from Bethel, Connecticut, had risen above his humble beginnings as an impoverished country boy and become a showman—indeed, the “greatest showman” (as he would claim) of his generation. Thanks to a combination of brilliant marketing tactics and less-than-upstanding business practices, Barnum had truly arrived, and with his book Humbugs of the World, published in 1865, he wanted to inform you, his audience, that he hadn’t achieved his rags-to-riches success story by scamming the public.

 

The use of parentheses in the underlined portion is most likely intended to

 

A. distinguish Barnum’s exact words from the surrounding description of his life.
B. emphasize the obstacles that Barnum overcame to achieve success.
C. illustrate the pride that Barnum took in his career.
D. suggest that Barnum may have exaggerated his accomplishments.

 

To answer this question, you must focus on the wording of the information in the parentheses. In context of the fact that Barnum was a showman given to less-than-upstanding business practices, the phrase as he would claim serves to emphasize the idea that Barnum was given to making over-the-top statements about his achievements, i.e., that he “may have exaggerated his accomplishments.” That makes the answer (D). (A) is incorrect because the information in the parentheses is not a direct quote. (B) is off-topic because the information in the parentheses has nothing to do with Barnum’s rags-to-riches story. Be careful with (C), though: it is obvious from the passage that Barnum took pride in his career, but the focus of the parenthetical information is on Barnum’s outsized claim about his accomplishments.

 


 

Looking for more practice? Check out the Quizzes and the Question of the Day Archives:

 

January
February
March
April
May
June
July
August
September
October
November
December

 

 

And for additional practice, Critical Reader Books include hundreds of practice questions and explanations.